You are on page 1of 70

APPLICATION OF DERIVATIVES

1.1. APPLICATION IN MECHANICS AND DY/DX AS A RATE MEASURE:

1.1.1. Velocity and Acceleration in Rectilinear Motion

The velocity of a moving particle is defined as the rate


V V+V
of change of its displacement with respect to time and s
s
the acceleration is defined as the rate of change of its
O A (t) B (t+t)
velocity with respect to time.
Let a particle A moves rectilinearly as shown in figure.
Let s be the displacement from a fixed point O along the path at time t; s is considered to be
positive on right of the point O and negative on the left of it.
Also, s is positive when s increases i.e., when the particle moves towards right.
s
Thus, if  s be the increment in s in time t . The average velocity in this interval is
t
s ds
And the instantaneous velocity i.e., velocity at time t is v  lim 
t 0 t dt
If the velocity varies, then there is change of velocity v in time t .
v dv
Hence, the acceleration at time t  lim 
t 0 t dt
1.1.2. Derivative as the Rate of Change

If a variable quantity y is some function of time t i.e., y  f (t ), then small change in time t
have a corresponding change y in y.
y
Thus, the average rate of change =
t
When limit t  0 is applied, the rate of change becomes instantaneous and we get the rate
of change with respect to t.
y dy
i.e., lim 
t dt
t  0

Hence, it is clear that the rate of change of any variable with respect to some other variable is
derivative of first variable with respect to other variable.
dy
Note :  The differential coefficient of y with respect to x i.e, is nothing but the rate of
dx
increase of y relative to x.

Nothing is impossible, the word itself says ‘I’m possible’!


2 APPLICATION OF DERIVATIVES

1.2. TANGENTS & NORMALS

1.2.1. Slope of the Tangent and Normal


(1) Slope of the tangent : If tangent is drawn on the curve y
Tangent
y  f (x ) at point P ( x1, y1 ) and this tangent makes an angle 
Norm al
with positive x-direction then,
 dy 
   tan = slope of the tangent
 dx  ( x1 , y1 )

x
O

 dy 
Note :  If tangent is parallel to x-axis   0    0
 dx ( x1 , y1 )
  dy 
 If tangent is perpendicular to x-axis     
2  dx ( x1 , y1 )

(2) Slope of the normal : The normal to a curve at P ( x1, y1 ) is a line perpendicular to the
tangent at P and passing through P and slope of the normal
1 1  dx 
= =   
Slope of tangent  dy   dy  P ( x1 , y1 )
 
 dx  P ( x1 , y1 )
 dx   dx 
Note :  If normal is parallel to x-axis      0 or   0
 dy  ( x1 , y1 )  dy  ( x1 , y1 )
 dy 
 If normal is perpendicular to x-axis (for parallel to y-axis)     0
 dx  ( x1 , y1 )

1.2.2. Equation of the Tangent and Normal

(1) Equation of the tangent : We know that the equation of a line passing through a point
P ( x1 , y1 ) and having slope m is y  y1  m( x  x1 )
 dy 
Slope of the tangent at ( x1 , y1 ) is =  
 dx  ( x1 , y1 )
The equation of the tangent to the curve y  f (x ) at point P ( x1 , y1 ) is

 dy 
y  y1    ( x  x1 )
 dx ( x1 , y1 )

CatalyseR Eduventures (India) Pvt. Ltd.


APPLICATION OF DERIVATIVES 3

1
(2) Equation of the normal : Slope of the Normal =
 dy 
 
 dx ( x1 , y1 )
Thus equation of the normal to the curve y  f (x ) at point P ( x1 , y1 )
1
y  y1  ( x  x1 )
 dy 
 
 dx  ( x1 , y1 )

Note :  If at any point P ( x1 , y1 ) on the curve y  f ( x ) , the tangent makes equal angle with the
 3 dy
axes, then at the point P,   or . Hence, at P, tan   1 .
4 4 dx

1.2.3. Angle of Intersection of Two Curves

The angle of intersection of two curves is defined to be the angle between the tangents to the
two curves at their point of intersection.
y
We know that the angle between two straight lines having slopes
y = f2 x
m1 and m2 y = f1 x

m1  m2
  tan 1
1  m1m2 P

Also slope of the tangent at P (x1 , y1 ) 

 dy   dy  x
m1    , m2    O
 dx 1( x1 , y1 )  dx 2( x1 , y1 )
Thus the angle between the tangents of the two curves y  f1 ( x ) and y  f 2 ( x )
 dy   dy 
 dx   
 1( x1 , y1 )  dx  2( x1 , y1 )
tan  
 dy   dy 
1    
 dx 1( x1 , y1 )  dx  2( x1 , y1 )

Orthogonal curves : If the angle of intersection of two curves is right angle, the two curves
are said to intersect orthogonally. The curves are called orthogonal curves.
 dy dy
If the curves are orthogonal, then   , m1m2  1       1
2  dx 1  dx  2

Nothing is impossible, the word itself says ‘I’m possible’!


4 APPLICATION OF DERIVATIVES

1.2.4. Length of Tangent, Normal, Subtangent and Subnormal


Let the tangent and normal at point P ( x, y) on the curve y  f (x ) meet the x-axis at points A
and B respectively. Then PA and PB are called length of tangent and normal respectively at
point P. If PC be the perpendicular from P on x-axis, then AC and BC are called length of
subtangent and subnormal respectively at P. If PA makes angle  with x-axis, then
dy
tan  from fig., we find that y
dx
2 Tangent
 dy  Norm al
1 
 dx 
(1) Length of tangent PA  ycosec  y
 dy  P (x, y)
 
 dx 

2 
 dy  x
(2) Length of normal PB  y sec   y 1    O A C B
 dx 
y
(3) Length of subtangent AC  y cot  
 dy 
 
 dx 
dy 
(4) Length of subnormal BC  y tan   y  
 dx 

1.2.5. Length of Perpendicular from Origin to the Tangent

Length of perpendicular from origin (0, 0) to the tangent drawn at point P ( x1 , y1 ) of the

 dy 
y1  x 1  
 dx  ( x1 , y1 )
curve y  f (x ) p
2
 dy 
1  
 dx 

CatalyseR Eduventures (India) Pvt. Ltd.


APPLICATION OF DERIVATIVES 5

1.3. MONOTONICITY (Significance of the sign of the first order derivative)


1.3.1. General Introduction

The most useful element taken into consideration amongst the total post mortuam
activities of functions, is their monotonic behaviour.
Functions are said to be monotonic if they are either increasing or decreasing in their
entire domain e.g. f (x) = ex ; f (x) = ln x & f (x) = 2x + 3 are some of the examples of

functions which are increasing whereas f (x) = – x3 ; f (x) = e–x and f (x) = cot–1(x) are some
of the examples of the functions which are decreasing.
Functions which are increasing as well as decreasing in their domain are said to be

non monotonic e.g. f (x) =sin x ; f (x) = ax2 + bx + c and f (x) = | x |, however in the interval
 
0, 2  , f (x) = sin x will be said to be increasing.

1.3.2. Monotonicity of A Function at A Point

A function is said to be monotonic increasing at x = a if f (x) satisfies


f  a  h   f  a  
 for a small positive h.......(1)
and f  a  h   f  a  
Small positive h means no discontinuity in f between a – h & a and a & a+h.

and monotonic decreasing at x = a if


f  a  h   f  a  
 .........(2)
and f  a  h   f  a  
It should be noted that we can talk of monotonocity of f(x) at x = a only if x = a lies in the
domain of f, without any consideration of continuity or differentiability of f (x) at x = a.

Nothing is impossible, the word itself says ‘I’m possible’!


6 APPLICATION OF DERIVATIVES

1.3.3. Monotonicity In An Interval

For an increasing function in some interval, if x > 0 y > 0 or x < 0  y < 0
then f is said to be monotonic (strictly) increasing in that interval. In other words if y and
dy dy
x have the same sign i.e. > 0, for increasing function. Hence if > 0 in some J
dx dx
(interval) then y is said to be increasing function in that J and conversely if f (x) is increasing
dy
in some J then > 0 in that J.
dx
dy
Similarly if < 0 in some J then y is decreasing in that J and conversely.
dx
dy
Hence to find the intervals of monotonocity for a function y = f (x) one has to find
dx
dy dy
and solve the inequality, > 0 or < 0. The solution of this inequality gives the
dx dx
interval of monotonocity.

It should however be noted that


dy
(a) at some point may be equal to zero but f (x) may still be increasing
dx
at x = a. Consider f (x) = x3 which is increasing at x = 0 although
f ' (x)=0. This is because f (0 + h) > f (0) and f (0 – h) < f (0). At all
dy
such points where = 0 but y is still increasing or decreasing are
dx
known as point of inflection, which indicate the change of concavity of
the curve.
(b) If f is increasing for x > 1 and f is also increasing for x < 1 then f is
also increasing as x = 1 provided f (1) is defined.

Similarly if f is decreasing for x > 1 and f is also decreasing for


x < 1 then f is also decreasing for x = 1 provided f (1) is defined.

However if f (1) is not defined then monotonocity will not be indicated at x = 1


1
e.g. f (x) = is decreasing for all x  R – {1}.
x 1
However if f is increasing and decreasing as shown then at
x =1 and x=2, f will have extremum values, being maximum at
x = 1 and minimum at x =2.

CatalyseR Eduventures (India) Pvt. Ltd.


APPLICATION OF DERIVATIVES 7

(c) If x1, x2  domain of f and if


(i) x1 > x2  f (x1) > f (x2), f is strictly increasing.
(ii) x1 > x2  f (x1)  f (x2), f is increasing.
Note that if a function is monotonic at x = a it can not have extremum point at x = a
and vice versa i.e. A point on the curve can not simultaneously be an extremum as well as
monotonic point.

1.3.4. Properties of Monotonic Function

(1) If f (x) is strictly increasing function on an interval [a, b], then f 1 exists and it is also a
strictly increasing function.

(2) If f (x) is strictly increasing function on an interval [a, b] such that it is continuous, then f 1
is continuous on [ f (a), f (b)]

(3) If f (x) is continuous on [a, b] such that f (c)  0( f (c)  0) for each c  (a, b), then f (x) is
monotonically (strictly) increasing function on [a, b].

(4) If f (x) is continuous on [a, b] such that f (c)  0( f (c)  0) for each c  (a, b) , then f (x) is
monotonically (strictly) decreasing function on [a, b]

(5) If f (x) and g(x ) are monotonically (or strictly) increasing (or decreasing) functions on [a, b],
then gof(x) is a monotonically (or strictly) increasing function on [a, b]

(6) If one of the two functions f (x) and g(x ) is strictly (or monotonically) increasing and other a
strictly (monotonically) decreasing, then gof(x) is strictly (monotonically) decreasing on
[a,b].

1.4. MAXIMA-MINIMA (Functions of Single Variable)


1.4.1. General Introduction

The notion of optimising functions is one of the most useful application of calculus used in
almost every sphere of life including geometry, business, trade, industries, economics,
medicines and even at home. In this chapter we shall see how calculus defines the notion of
maxima and minima and distinguishes it from the greatest and least value or global maxima
and global minima of a function. Since most of the functions which we encounter with in
practical world are differentiable hence we continue our discussion with such functions only
unless otherwise stated.

Nothing is impossible, the word itself says ‘I’m possible’!


8 APPLICATION OF DERIVATIVES

1.4.2. How Maxima & Minima Are Classified

1. A function f (x) is said to have a maximum at x = a if


f (a) is greater than every other value assumed by f (x)
in the immediate neighbourhood of x = a. Symbolically
f  a   f  a  h  
  x = a gives maxima
f  a   f  a  h  
for a sufficiently small positive h.
Similarly, a function f (x) is said to have a minimum value at x = b if f (b) is least than every
other value assumed by f (x) in the immediate neighbourhood at x = b. Symbolically if
f  b   f  b  h  
  x = b gives minima for a sufficiently small positive h.
f  b   f  b  h  
Note that :
(i) the maximum & minimum values of a function are also
known as local/relative maxima or local/relative minima as
these are the greatest & least values of the function relative
to some neighbourhood of the point in question.

(ii) the term 'extremum' or (extremal) or 'turning value' is used


both for maximum or a minimum value.

(iii) a maximum (minimum) value of a function may not be the greatest (least) value in a finite
interval.
(iv) a function can have several maximum & minimum values & a minimum value may even be
greater than a maximum value.
(v) maximum & minimum values of a continuous function occur alternately & between two
consecutive maximum values there is a minimum value & vice versa.

1.4.3. Necessary And Sufficient Conditions for Maximum & Minimum


dy
Consider the interval (a – h, a), we find f (x) is increasing  > 0. Similarly for the
dx
dy
interval (a, a + h), we find f (x) is decreasing  < 0.
dx
dy
Hence at the point x = a (maxima) ; =0
dx
dy
similarly = 0 at x = b which is the point of minima.
dx

CatalyseR Eduventures (India) Pvt. Ltd.


APPLICATION OF DERIVATIVES 9

dy
Hence = 0 is the necessary condition for maxima or
dx
dy
minima. These points where vanishes are known as
dx
stationary points as instantaneous rate of change of function
momentarily ceases at this point.
However
dy dy
If for x  a,  0 and for x  a,  0  x  a is a point of local maxima
dx dx
dy dy
and if for x  b,  0 and for x  b,  0  x  b is a point of local minima
dx dx
Hence if f  (a  h) > 0 and f  (a  h) < 0  x = a is a point of local maxima, where f  (a) = 0.
Similarly f  (b  h)  0 and f  (b  h)  0  x = b is a point of local minima, where f  (b) = 0.
However, if f  (x) does not change sign i.e. has the same sign in a certain complete
neighbourhood of c, then f(x) is either strictly increasing or decreasing throughout this
neighbourhood implying that f(c) is not an extreme value of f. e.g. f (x) = x3 at x = 0.

1.4.4. Use of Second Order Derivative in Ascertaining The Maxima or Minima

dy
As shown in the figure it is clear that as x increases from a – h to a + h, the function
dx
dy
continuously decreases, i.e. (+) ve for x < a, zero at x = a and (–) ve for x > a. Hence
dx
d2y
itself is a decreasing function. Therefore < 0 in (a – h, a + h).
dx2
dy d2y
Hence at local maxima, = 0 and < 0.
dx dx2
dy d2y
similarly at local minima, = 0 and > 0.
dx dx2
Hence if
(a) f (a) is a maximum value of the function f then f ' (a) = 0 & f " (a) < 0.
(b) f (b) is a minimum value of the function f, if f ' (b) = 0 & f " (b) > 0.
However, if f " (c) = 0 then the test fails. In this case f can still have a maxima or minima
or point of inflection (neither maxima nor minima). In this case revert back to the first order
derivative check for ascertaning the maxima or minima.

Nothing is impossible, the word itself says ‘I’m possible’!


10 APPLICATION OF DERIVATIVES

1.4.5. Summary – Working Rule

FIRST : When possible, draw a figure to illustrate the problem & label those parts that
are important in the problem. Constants & variables should be clearly distinguished.

SECOND : Write an equation for the quantity that is to be maximised or minimised. If


this quantity is denoted by ‘y’, it must be expressed in terms of a single
independent variable x. This may require some algebraic manipulations.

THIRD : If y = f (x) is a quantity to be maximum or minimum, find those values of x for


which dy/dx = f (x) = 0.

FOURTH : Test each values of x for which f (x) = 0 to determine whether it provides a
maximum or minimum or neither. The usual tests are :
(a) If d²y/dx² is positive when dy/dx = 0  y is minimum.
If d²y/dx² is negative when dy/dx = 0  y is maximum.
If d²y/dx² = 0 when dy/dx = 0, the test fails.
positive for x  x0 
dy
(b) If is zero for x  x0  a maximum occurs at x = x0.
dx
negative for x  x0 
But if dy/dx changes sign from negative to zero to positive as x advances through
x0 there is a minimum . If dy/dx does not change sign, neither a maximum nor a
minimum. Such points are called INFLECTION POINTS.

FIFTH : If the function y = f (x) is defined for only a limited range of values a  x  b then
examine x = a & x = b for possible extreme values.

SIXTH : If the derivative fails to exist at some point, examine this point as possible maximum
or minimum.

1.4.6. Properties of Maxima and Minima

(i) If f (x) is continuous function in its domain, then at least one maxima and one minima must
lie between two equal values of x.
(ii) Maxima and minima occur alternately, that is, between two maxima there is one minimum
and vice-versa.
(iii) If f (x )   as x  a or b and f ( x )  0 only for one value of x (say c) between a and b, then
f (c) is necessarily the minimum and the least value.
If f (x )   as x  a or b, then f (c) is necessarily the maximum and the greatest value.

CatalyseR Eduventures (India) Pvt. Ltd.


APPLICATION OF DERIVATIVES 11

1.4.7. Greatest and Least Values of a Function in a given Interval


If a function f (x) is defined in an interval [a, b], then greatest or least values of this function
occurs either at x  a or x  b or at those values of x where f ( x )  0 .
Remember that a maximum value of the function f (x) in any interval [a, b] is not necessarily
its greatest value in that interval.
Thus greatest value of f(x) in interval [a, b] = max. [ f (a), f (b), f (c)]
Least value of f (x) interval [a, b]= min. [ f (a), f (b), f (c)]
Where x  c is a point such that f (c)  0

1.4.8. Geometrical Results related to Maxima and Minima


The following results can easily be established.
(1) The area of rectangle with given perimeter is greatest when it is a square.
(2) The perimeter of a rectangle with given area is least when it is a square.
(3) The greatest rectangle inscribed in a given circle is a square.
(4) The greatest triangle inscribed in given circle is equilateral.
(5) The semi vertical angle of a cone with given slant height and maximum volume is tan 1 2
(6) The height of a cylinder of maximum volume inscribed in a sphere of radius a is a 2a / 3 .

1.4.9. Significance of The Sign of 2nd Order Derivative and Points of Inflection

The sign of the 2nd order derivative determines the concavity of the
curve. Such points such as C & E on the graph where the concavity
of the curve changes are called the points of inflection. From the
graph we find that if:
d2y
(i) > 0  concave upwards
dx2
d2y
(ii) < 0  concave downwards.
dx2
d2y d2y
At the point of inflection we find that =0& changes sign.
dx2 dx2
d2y
Inflection points can also occur if fails to exist . For example,
dx2
consider the graph of the function defined as,
 x3/5 for x  (,1)
f (x) =  2
 2x for x  (1, )
Note that the graph exhibits two critical points one is a point of local
maximum & the other a point of inflection.

Nothing is impossible, the word itself says ‘I’m possible’!


12 APPLICATION OF DERIVATIVES

1.5. ROLLE’S & MEAN VALUE THEOREM

1.5.1. Rolle’s Theorem

Let f(x) be a function of x subject to the following conditions :


(i) f(x) is a continuous function of x in the closed interval of a  x  b.
(ii) f  (x) exists for every point in the open interval a < x < b.
(iii) f (a) = f (b).
Then there exists at least one point x = c such that a < c < b where f  (c) = 0.
Note that if f is not continuous in closed [a, b] then it may lead to the adjacent graph where
all the 3 conditions of Rolles will be valid but the assertion will not be true in (a, b).

EXPLANATION :
Note : Converse of Rolle's theorem is Not true i.e. if a function is such that
f ' (c) = 0 for at least one 'c' in (a, b) then it is not necessary
(i) f (x) is continuous in [a, b]
(ii) f (x) is differentiable in (a, b)
(iii) f (a) = f (b)
1.5.2. LMVT Theorem (Lagrange’s Mean Value Theorem)

Let f(x) be a function of x subject to the following conditions :


(i) f(x) is a continuous function of x in the closed interval of a  x  b.
(ii) f  (x) exists for every point in the open interval a < x < b.
(iii) f(a)  f(b).
Then there exists at least one point x = c such that a < c < b where
f (b)  f (a)
f  (c) = Geometrically, the slope of the secant line joining the curve at x = a
ba
& x = b is equal to the slope of the tangent line drawn to the curve at x = c. Note the
following :
Note : Now [f (b) – f (a)] is the change in the value of function f as x changes from a to b
so that [f (b) – f (a)] / (b – a) is the average rate of change of the function over the interval
[a, b]. Also f '(c) is the instantaneous rate of change of the function at x = c. Thus, the
theorem states that the average rate of change of a function over an interval is also the actual
rate of change of the function at some point of the interval. In particular, for instance, the
average velocity of a particle over an interval of time is equal to the velocity at some instant
belonging to the interval.

CatalyseR Eduventures (India) Pvt. Ltd.


APPLICATION OF DERIVATIVES 13

This interpretation of the theorem justifies the name "Mean Value" for the theorem.

Rolles theorem is a special case of LMVT since


f (b)  f (a)
f (a) = f (b)  f  (c) = = 0.
ba
Alternative form of LMVT :
Another form of statement of Lagrange's Mean Value Theorem. If a function f is
continuous in a closed interval [a, a + h] and derivable in the open interval ] a, a + h [, then
there exists at least one number ''  (0, 1) such that f (a + h) = f (a) + h f ' (a + h).

1.6. SOLVED EXAMPLE

1. The distance travelled s (in metre) by a particle in t second is given by s  t 3  2t 2  t .


The speed of the particle after 1 sec. will be
(a) 8 cm/sec. (b) 6 cm/sec. (c) 2 cm/sec (d) None of these
ds
Solution: (a) s  t 3  2t 2  t , v   3t 2  4t  1 Speed of the particle after 1 second
dt
 ds 
v(t 1)     3 12  4 1  1  8cm / sec.
dt
 (t 1)

2. If the volume of a spherical balloon is increasing at the rate of 900 cm2/sec. then the
rate of change of radius of balloon at instant when radius is 15 cm [in cm/sec]
22 7
(a) (b) 22 (c) (d) None of these
7 22
4
Solution: (c) V   r 3 Differentiate with respect to t
3
dV 4 dr dr 1 dV dr 1 1 7
  3r 2 .   2
.   900   .
dt 3 dt dt 4 r dt dt 4    15  15  22

3. The line x  y  2 is tangent to the curve x 2  3  2 y at its point


(a) (1, 1) (b) (–1, 1) (c) ( 3 , 0) (d) (3, – 3)
2 dy dy
Solution: (a) Given curve x 2  3  2 y diff. w.r.t. x, 2 x   ;  x
dx dx
Slope of the line = – 1
dy
  x  1 ; x  1
dx

 y 1 point (1, 1)

Nothing is impossible, the word itself says ‘I’m possible’!


14 APPLICATION OF DERIVATIVES

4. The tangent to the curve y  2 x 2  x  1 at a point P is parallel to y  3x  4, the co-


ordinate of P are
(a) (2, 1) (b) (1, 2) (c) (– 1, 2) (d) (2, – 1)
Solution: (b) Given y  2x 2  x  1

 dy 
Let the co-ordinate of P is (h, k) then    4h  1
 dx ( h, k )
Clearly 4h  1  3 h  1  k  2 . P is (1, 2).

5. The angle between the curves y 2  x and x 2  y at (1, 1) is


4 3
(a) tan 1 (b) tan 1 (c) 90o (d) 45o
3 4
dy dy
Solution: (b) Given curve y 2  x and x 2  y Differentiating w.r.t. x, 2 y  1 and 2 x 
dx dx
 dy  1  dy 
   and    2 Angle between the curve
 dx (1,1) 2  dx (1,1)
1
2
 tan   2  tan   3    tan 1 3 .
1 4 4
1  .2
2

2
6. On the interval (1, 3) the function f ( x)  3 x  is
x
(a) Strictly decreasing (b) Strictly increasing
(c) Decreasing in (2, 3) only (d) Neither increasing nor decreasing
2 2
Solution: (b) f ( x)  3 x   f ( x)  3  2 Clearly f ( x )  0 on the interval (1, 3)
x x
 f ( x ) is strictly increasing.

ln(  x)
7. The function f ( x )  is
ln(e  x)
(a) Increasing on [0, )
(b) Decreasing on [0, )
   
(c) Decreasing on  0,  and increasing on  ,  
 e e 
   
(d) Increasing on  0,  and decreasing on  e ,  
 e

CatalyseR Eduventures (India) Pvt. Ltd.


APPLICATION OF DERIVATIVES 15

ln(  x)
Solution: (b) Let f ( x ) 
ln(e  x)
1 1
ln(e  x)   ln(  x )
 f ( x)   x e  x = (e  x) ln(e  x )  (  x ) ln(  x)
2
ln ( e  x) {ln(e  x)}2  (e  x )(  x)
 f ( x)  0 for all x  0 {  e} . Hence, f ( x ) is decreasing in [0,  ) .

8. The interval in which the function x 2 e  x is non decreasing, is


(a) ( , 2] (b) [0, 2] (c) [2,  ) (d) None of these
dy
Solution: (b) Let f ( x )  x 2e  x   2 xe  x  x 2 e  x  e  x (2 x  x 2 )
dx
Hence f ( x)  0 for every x  [0, 2] , therefore it is non-decreasing in [0, 2].

9. The maximum value of sin x(1  cos x) will be at


  
(a) x  (b) x  (c) x  (d) x  
2 6 3
1
Solution: (c) y  sin x(1  cos x )  sin x  sin 2 x
2
dy d2y
  cos x  cos 2 x and   sin x  2sin 2 x
dx dx 2
dy
On putting  0, cos x  cos 2 x  0  cos x   cos 2 x  cos(  2 x )  x    2 x
dx
  d2y  1  2   3 3 3 3
 x ,   2   sin     2sin    =  2. 
3  dx  x /3 3  3  2 2 2

which is negative.  at x 
the function is maximum.
3
10. The adjacent sides of a rectangle with given perimeter as 100 cm and enclosing
maximum area are
(a) 10 cm and 40 cm (b) 20 cm and 30 cm
(c) 25 cm and 25 cm (d) 15 cm and 35 cm
Solution: (c) 2 x  2 y  100  x  y  50 .....(i)
A
Let area of rectangle is A,  A  xy  y 
x
A dA dA
From (i), x   50  A  50 x  x 2   50  2 x for maximum area 0
x dx dx
 50  2 x  0  x  25 and y  25  adjacent sides are 25 cm and 25 cm.

Nothing is impossible, the word itself says ‘I’m possible’!


16 APPLICATION OF DERIVATIVES

11. The function f ( x )  x ( x  3)e 1/ 2 x satisfies all the condition of Rolle's theorem in
[– 3, 0]. The value of c is
(a) 0 (b) 1 (c) – 2 (d) – 3
Solution: (c) To determine 'c' in Rolle's theorem, f (c)  0
 1  1 
Here f ( x)  ( x 2  3 x)e  (1/2) x .     (2 x  3)e  (1/2) x = e (1/2) x  ( x 2  3 x)  2 x  3
 2  2 
1
=  e  ( x / 2)  x 2  x  6 2
 f (c )  0  c  c  6  0  c  3,  2 .
2
But c  3  [ 3, 0] , Hence c = –2.

12. The two curves x3 - 3xy2 + 5 = 0 and 3x2 y - y3 - 7 = 0


(a) cut at right angles (b) touch each other
p p
(c) cut at an angle (d) cut at an angle
4 3
SOLUTION: (a) Differentiating x3 - 3xy2 + 5 = 0 ,
dy dy x 2  y 2
we get 3x 2  3y 2  6xy 0 
dx dx 2xy
Differentiating 3x2 y - y3 - 7 = 0 , we get
dy dy dy 2xy
6xy  3x2  3y2 0   2
dx dx dx y  x2
x2 - y2 2xy
Since, product of slopes is ´ 2 = - 1  The two curves cut at right angle.
2xy y - x2
13. The number of values of k for which the equation x 3 - 3x + k = 0 has two different
roots lying in the interval (0, 1) are
(a) 3 (b) 2
(c) infinitely many (d) no value of k satisfies the requirement.
SOLUTION: (d) Let f (x ) = x 3 - 3x + k . Let, if possible, a, b Î (0, 1) such that
f 'c  0  3c 2  3  0  c  1  no value of k satisfies the condition.
14. If f and g are two increasing functions such that gof is defined, then
(a) gof is an increasing function (b) gof is a decreasing function
(c) gof is neither increasing nor decreasing (d) None of these
SOLUTION: (a) Let x1, x 2  R such that x1 < x 2  f  x1   f  x 2  ( f is increasing)

 g  f  x1    g  f  x 2   ( g is increasing)
  gof  x1    gof  x 2   gof is increasing.

CatalyseR Eduventures (India) Pvt. Ltd.


APPLICATION OF DERIVATIVES 17

15. A rod of length 13 metres has one end P on the x-axis and the other end Q on the
y-axis. If P moves on the x-axis with the speed of 12 m/sec, then the speed of the
other end Q when it is 12 m from the origin is
(a) 3 m/sec (b) 5 m/sec (c) –5 m/sec (d) 4 m/sec
2
SOLUTION: (c) x 2 + y 2 = (13 ) = 169
dx dy
 2x  2y 0 Q
dt dt
dx dy dx 13 m
 x y 0 = 12 (given) y
dt dt dt
dx dy
 x y 0 x P
dt dt x
But, y = 12 (given)
 x 2  144  169  x 2  25  x  5
dy 12  5 
   5  speed of Q = –5 m/sec.
dt y 12 12



Nothing is impossible, the word itself says ‘I’m possible’!


18 APPLICATION OF DERIVATIVES

1.7. ADVANCE OBJECTIVE EXERCISE

ADVANCE OBJECTIVE EXERCISE - 1

b  2 x , if x  1
Q.1 Let f : R  R be defined by f(x) = 
2 x  3 , if x  1
If f has a local minimum at x = – 1, then a possible value of b is equal to
1
(a) 0 (b) (c) – 1 (d) 1
2
x

Q.2 The angle between the tangent lines to the graph of the function f (x) =  (2t  5) dt
2
at the

points where the graph cuts the x-axis is


   
(a) (b) (c) (d)
6 4 3 2

Q.3 If a < b < c < d & x  R then the least value of the function,
f(x) = x  a + x  b + x  c + x  d is
(a) c – d + b – a (b) c + d – b – a (c) c + d – b + a (d) c – d + b + a

Q.4 If a variable tangent to the curve x2 y = c3 makes intercepts a, b on x and y axis respectively,
then the value of a2b is
4 3 27 3 4 3
(a) 27 c3 (b) c (c) c (d) c
27 4 9

1 1 1
Q.5 Let f (x) = 3 x 5  3x 2 3
3 x  1 . Then the equation f (x) = 0 has
2 x  1 3x  1 7 x8  1
2 5

(a) no real root


(b) atmost one real root
(c) atleast 2 real roots
(d) exactly one real root in (0,1) and no other real root.

Q.6 Number of roots of the equation x2 – 2x – log2 | 1 – x | = 3 is


(a) 4 (b) 2 (c) 1 (d) 0

Q.7 Difference between the greatest and the least values of the function
f (x) = x(ln x – 2) on [1, e2] is
(a) 2 (b) e (c) e2 (d) 1

CatalyseR Eduventures (India) Pvt. Ltd.


APPLICATION OF DERIVATIVES 19

Q.8
2 2

The intervals of increase of f(x) defined by f ( x)   t  2t t 1 dt   is equal to
1

 3   1 1 
(a)  ,   (0, 3)  (10, ) (b) (– , – 2)   ,   (4, )
 2   2 2
 3 1 
(c) (– , – 2)  (– 1, 0)  (1, ) (d) (– , – 2)   ,   (1, )
 4 4

Q.9 If the normal of y = f(x) at (0, 0) is given by y – x = 0, then


x2
Lim is equal to
x  0 f ( x2 )  20 f (9 x2 )  2 f (99 x 2 )

1 1 1
(a) (b) (c) (d) non-existent
19 19 2

Q.10 Number of positive integral values of ‘a’ for which the curve y = a x intersects the line y = x
is
(a) 0 (b) 1 (c) 2 (d) More than 2

Q.11 Let C be the curve y = x3 (where x takes all real values). The tangent at A meets the curve
again at B. If the gradient at B is K times the gradient at A then K is equal to
1
(a) 4 (b) 2 (c) – 2 (d)
4

3 2
Q.12 The lines y =  x and y =  x intersect the curve P(x1,y1) 2
(0, — )
2 5 Ö5
Q (x2,y2) 2

3x2 + 4xy + 5y2  4 = 0 at the points P and Q respectively. The 0
Ö3
2
–—
tangents drawn to the curve at P and Q : Ö3
2
(0,– — )
(a) intersect each other at angle of 45º (b) are parallel to each other Ö5

(c) are perpendicular to each other (d) none of these

Q.13 The bottom of the legs of a three legged table are the vertices of an isoceles triangle with
sides 5, 5 and 6. The legs are to be braced at the bottom by three wires in the shape of a Y.
The minimum length of the wire needed for this purpose, is
(a) 4 + 3 3 (b) 10 (c) 3 + 4 3 (d) 1 + 6 2

Nothing is impossible, the word itself says ‘I’m possible’!


20 APPLICATION OF DERIVATIVES

x 1
Q.14 If Mean value theorem holds good for the function f (x) = on the interval [1, 3] then
x
the value of 'c' is
1 2
(a) 2 (b) (c) (d) 3
3 3
2
Q.15 Let g : [1, 6]  [0, ) be a real valued differentiable function satisfying g(x) =
x  g ( x)
and g(1) = 0, then the maximum value of g cannot exceed
(a) ln 2 (b) ln 6 (c) 6 ln 2 (d) 2 ln 6

ANSWER KEY

1 2 3 4 5 6 7 8 9 10

C D B C C A B C B B

11 12 13 14 15

A C A D D

CatalyseR Eduventures (India) Pvt. Ltd.


APPLICATION OF DERIVATIVES 21

ADVANCE OBJECTIVE EXERCISE - 2

Q.1 If the function f (x) = x4 + bx2 + 8x + 1 has a horizontal tangent and a point of inflection for
the same value of x then the value of b is equal to
(a) – 1 (b) 1 (c) 6 (d) – 6

Q.2 Coffee is draining from a conical filter, height and diameter both 15 cms into a cylinderical
coffee pot diameter 15 cm. The rate at which coffee drains from the filter into the pot is 100
cu cm /min.
The rate in cms/min at which the level in the pot is rising at the instant when the coffee in the
pot is 10 cm, is
9 25 5 16
(a) (b) (c) (d)
16 9 3 9

Q.3 A horse runs along a circle with a speed of 20 km/hr . A lantern is at the centre of the circle.
A fence is along the tangent to the circle at the point at which the horse starts . The speed
with which the shadow of the horse moves along the fence at the moment when it covers 1/8
of the circle in km/hr is
(a) 20 (b) 40 (c) 30 (d) 60

Q.4 Let a, b, c, d are non-zero real numbers such that 6a + 4b + 3c + 3d = 0, then the equation
ax3 + bx2 + cx + d = 0 has
(a) atleast one root in [–2, 0] (b) atleast one root in [0, 2]
(c) atleast two roots in [–2, 2] (d) no root in [–2, 2]

Q.5 Give the correct order of initials T or F for following statements. Use T if statement is true
and F if it is false.
Statement-1: If f : R  R and c  R is such that f is increasing in (c – , c) and f is
decreasing in
(c, c + ) then f has a local maximum at c. Where  is a sufficiently small positive quantity.
Statement-2 : Let f : (a, b)  R, c  (a, b). Then f can not have both a local maximum and a
point of inflection at x = c.
Statement-3 : The function f (x) = x2 | x | is twice differentiable at x = 0.
Statement-4 : Let f : [c – 1, c + 1]  [a, b] be bijective map such that f is differentiable at c
then f–1 is also differentiable at f (c) .
(a) FFTF (b) TTFT (c) FTTF (d) TTTF

Nothing is impossible, the word itself says ‘I’m possible’!


22 APPLICATION OF DERIVATIVES

Q.6 The lateral edge of a regular hexagonal pyramid is 1 cm. If the volume is maximum, then its
height must be equal to :
1 2 1
(a) (b) (c) (d) 1
3 3 3

Q.7 In a regular triangular prism the distance from the centre of one base to one of the vertices of
the other base is l . The altitude of the prism for which the volume is greatest :
l l l l
(a) (b) (c) (d)
2 3 3 4

Q.8 The curve y  exy + x = 0 has a vertical tangent at :


(a) (1, 1) (b) (0, 1) (c) (1, 0) (d) no point

Q.9 Number of solution(s) satisfying the equation, 3x2  2x3 = log2 (x2 + 1)  log2 x is :
(a) 1 (b) 2 (c) 3 (d) none

Q.10 Consider the function f (x) = x cos x – sin x, then identify the statement which is correct .
(a) f is neither odd nor even (b) f is monotonic decreasing at x = 0
(c) f has a maxima at x =  (d) f has a minima at x = – 

Q.11 Let f (x) = x3 – 3x2 + 2x. If the equation f (x) = k has exactly one positive and one negative
solution then the value of k equals
2 3 2 2 1
(a) – (b) – (c) (d)
9 9 3 3 3 3

a b
Q.12 The x-intercept of the tangent at any arbitrary point of the curve  = 1 is proportional
x2 y 2
to:
(a) square of the abscissa of the point of tangency
(b) square root of the abscissa of the point of tangency
(c) cube of the abscissa of the point of tangency
(d) cube root of the abscissa of the point of tangency .

Q.13 The function 'f' is defined by f(x) = xp (1  x)q for all x  R, where p,q are positive
integers, has a maximum value, for x equal to :
pq p
(a) (b) 1 (c) 0 (d)
pq pq

CatalyseR Eduventures (India) Pvt. Ltd.


APPLICATION OF DERIVATIVES 23

Q.14 Let h be a twice continuously differentiable positive function on an open interval J. Let
g(x) = ln  h ( x)  for each x  J
2
Suppose  h '( x )  > h''(x) h(x) for each x  J. Then
(a) g is increasing on J (b) g is decreasing on J
(c) g is concave up on J (d) g is concave down on J

Q.15 Let f (x) and g (x) be two continuous functions defined from R  R, such that f (x1) > f

(x2) and g (x1) < g (x2),  x1 > x2 , then solution set of f  g ( 2  2 )  > f  g (3  4) 
is
(a) R (b)  (c) (1, 4) (d) R – [1, 4]

ANSWER KEY

1 2 3 4 5 6 7 8 9 10

D D B B C C B C A B

11 12 13 14 15

A C D D C

ADVANCE OBJECTIVE EXERCISE - 3

1
Q.1 Let f (x) = x – then which one of the following statement is true
x
(a) Function is invertible if defined from R – {0}  R.
(b) f (x1) > f (x2),  x1 > x2 and x1 ,x2  0.
(c) Graph of the function has exactly one asymptote.
(d) Function is one-one in every continuous interval [a, b] defined on one side of origin.

t  3x  x 2
Q.2 If the function f (x) = , where 't' is a parameter has a minimum and a maximum
x4
then the range of values of 't' is
(a) (0, 4) (b) (0, )
(c) (– , 4) (d) (4, )

Nothing is impossible, the word itself says ‘I’m possible’!


24 APPLICATION OF DERIVATIVES

Q.3 A point is moving along the curve y3 = 27x. The interval in which the abscissa changes at
slower rate than ordinate, is
(a) (–3 , 3) (b) (–  ,  ) (c) (–1, 1) (d) (– , –3)(3, )

Q.4 The graph of y = f (x) is shown. Let F (x) be an antiderivative of f (x). Then F(x) has
y
y=f(x)

2/3 4 /3
x
/2  3/2 2

2 4 
(a) points of inflexion at x = 0, , , and 2, a local maximum at x = , and a local
3 3 2
3
minimum at x =
2
2 4 
(b) points of inflexion at x = 0, , , and 2, a local minimum at x = , and a local
3 3 2
3
maximum at x =
2
 3
(c) point of inflexion at x = , a local maximum at x = , and a local minimum at x =
2 2
 3
(d) point of inflexion at x = , a local minimum at x = , and a local maximum at x =
2 2

Q.5 P and Q are two points on a circle of centre C and radius , the angle PCQ being 2 then the
radius of the circle inscribed in the triangle CPQ is maximum when
3 1 5 1 5 1 5 1
(a) sin   (b) sin   (c) sin   (d) sin  
2 2 2 2 4

x
2 x
  12  12 cos 2 t  t 
3
Q.6 Number of critical points of the function, f(x) = x   dt
3 2 1

which lie in the interval [ 2, 2] is


(a) 2 (b) 4 (c) 6 (d) 8

x
Q.7 The range of values of m for which the line y = mx and the curve y = 2
enclose a
x 1
region, is
(a) (–1, 1) (b) (0, 1) (c) [0, 1] (d) (1, )

CatalyseR Eduventures (India) Pvt. Ltd.


APPLICATION OF DERIVATIVES 25

Q.8 For a steamer the consumption of petrol (per hour) varies as the cube of its speed (in km). If
the speed of the current is steady at C km/hr then the most economical speed of the steamer
going against the current will be
(a) 1.25 C (b) 1.5 C
(c) 1.75C (d) 2 C
a x3
Q.9 The set of value(s) of 'a' for which the function f (x) = + (a + 2) x2 + (a  1) x + 2
3
possess a negative point of inflection .
(a) (,  2)  (0, ) (b) { 4/5 }
(c) ( 2, 0) (d) empty set

Q.10 The number of points of local maximum and local minimum of the function
x2
t 2  5t  4
f(x) =  dt are respectively
0
2  et
(a) 2, 2 (b) 3, 2
(c) 2, 3 (d) 1, 1

Q.11 Let f (x) = x + x on [1, 4]. The mean value theorem says that there must be some number
'c' between 1 and 4 so that f '(c) is equal to the average slope of f (x) on [1, 4]. the number
'c' must be
5 9
(a) (b)
2 4
11
(c) (d) 3
4

Q.12 Read the following mathematical statements carefully:


I. Adifferentiable function ' f ' with maximum at x = c  f ''(c) < 0.
II. Antiderivative of a periodic function is also a periodic function.
T T

III. If f has a period T then for any a  R.  f ( x) dx =  f ( x  a) dx


0 0

IV. If f (x) has a maxima at x = c , then 'f ' is increasing in (c – h, c) and decreasing in
(c, c + h) as h  0 for h > 0.
Now indicate the correct alternative.
(a) exactly one statement is correct. (b) exactly two statements are correct.
(c) exactly three statements are correct. (d) All the four statements are correct.

Nothing is impossible, the word itself says ‘I’m possible’!


26 APPLICATION OF DERIVATIVES

Q.13 If the point of minima of the function, f(x) = 1 + a2x – x3 satisfy the inequality
x2  x  2
< 0, then 'a' must lie in the interval:
x2  5x  6
(a)  3 3, 3 3  (b)  2 3,  3 3 
(c)  2 3, 3 3  (d)  3 3,  2 3    2 3, 3 3 
Q.14 Two sides of a triangle are to have lengths 'a' cm & 'b' cm. If the triangle is to have the
maximum area, then the length of the median from the vertex containing the sides 'a' and 'b'
is

1 2 2a  b a 2  b2 a  2b
(a) a  b2 (b) (c) (d)
2 3 2 3

Q.15 A rectangle has one side on the positive y-axis and one side on the positive x - axis. The
nx
upper right hand vertex of the rectangle lies on the curve y = . The maximum area of the
x2
rectangle is
(a) e–1 (b) e – ½ (c) 1 (d) e½



ANSWER KEY

1 2 3 4 5 6 7 8 9 10

D C C C B B B B A C

11 12 13 14 15

B A D A A

CatalyseR Eduventures (India) Pvt. Ltd.


APPLICATION OF DERIVATIVES 27

ADVANCE OBJECTIVE EXERCISE - 4

Q.1 A particle moves along the curve y = x3/2 in the first quadrant in such a way that its distance
dx
from the origin increases at the rate of 11 units per second. The value of when x = 3 is
dt
9 3 3
(a) 4 (b) (c) (d) none
2 2

  
Q.2 Number of solution of the equation 3tanx + x3 = 2 in  0,  is
 4
(a) 0 (b) 1 (c) 2 (d) 3


Q.3 Consider f (x) = | 1 – x | 1  x  2 and g (x) = f (x) + b sin x, 1<x<2
2
then which of the following is correct?
3
(a) Rolles theorem is applicable to both f, g and b =
2
1
(b) LMVT is not applicable to f and Rolles theorem if applicable to g with b =
2
(c) LMVT is applicable to f and Rolles theorem is applicable to g with b = 1
(d) Rolles theorem is not applicable to both f, g for any real b.

Q.4 The angle made by the tangent of the curve x = a (t + sint cost) ; y = a (1 + sint)2 with the x-
axis at any point on it is
1 1  sin t 1 1  sin t
(a)   2t  (b) (c)  2t    (d)
4 cos t 4 cos 2t

Q.5 If f be a continuous function on [0, 1], differentiable in (0, 1) such that f (1) = 0, then their
exists some c  (0, 1) such that
(a) c f ' (c) – f (c) = 0 (b) f ' (c) + c f (c) = 0
(c) f ' (c) – c f (c) = 0 (d) c f ' (c) + f (c) = 0

Q.6 If the function f (x) = 2x2 + 3x + 5 satisfies LMVT at x = 2 on the closed interval [1, a] then
the value of 'a' is equal to
(a) 3 (b) 4 (c) 6 (d) 1

Q.7 Consider the function f (x) = 8x2 – 7x + 5 on the interval [–6, 6]. The value of c that satisfies
the conclusion of the mean value theorem, is
(a) – 7/8 (b) – 4 (c) 7/8 (d) 0

Nothing is impossible, the word itself says ‘I’m possible’!


28 APPLICATION OF DERIVATIVES


2

  sin x  t cos x  dx
0
Q.8 The value of t for which is maximum lies in the interval

2
2
  sin x  t cos x 
0
dx

  1 3
(a)  0,  (b)  , 
 2 2 4
3 3 3 5
(c)  ,  (d)  , 
4 2 2 2

Q.9 Suppose that f is differentiable for all x and that f '(x)  2 for all x. If f (1) = 2 and f (4) = 8
then f (2) has the value equal to
(a) 3 (b) 4 (c) 6 (d) 8

Q.10 There are 50 apple trees in an orchard. Each tree produces 800 apples. For each additional
tree planted in the orchard, the output per additional tree drops by 10 apples. Number of trees
that should be added to the existing orchard for maximising the output of the trees, is
(a) 5 (b) 10
(c) 15 (d) 20

1
Q.11 Which of the following statements is/are always true concerning the function f (x) = x +
x
I The graph of the function is symmetric about the y-axis
II The graph of the function has a relatively maximum at (1, 2) and a relative minimum at
(–1, –2)
III The graph of the function is concave up for x > 0 and concave down for x < 0.
(a) only I (b) only II (c) only III (d) only I and II

nx
Q.12 Range of the function f(x) = is
x
 2  1
(a) (–, e) (b) (–, e2) (c)  ,  (d)  , 
 e  e

Q.13 If M (x0, y0) is the point on the curve 3x2 – 4y2 = 72, which is nearest to the line
3x + 2y + 1 = 0, then the value of (x0 + y0) is equal to
(a) 3 (b) – 3 (c) 9 (d) – 9

CatalyseR Eduventures (India) Pvt. Ltd.


APPLICATION OF DERIVATIVES 29

Q.14 The equation of normal to the curve x + y = xy, where it cuts the x-axis is equal to
(a) y = – 2x + 2 (b) y = 3x – 3 (c) y = x – 1 (d) 2y = x – 1

Q.15 If f(x) = x3 + ax2 + bx + 5 sin2 x is a strictly increasing function on the set of real numbers
then a and b must satisfy the relation
(a) a2 – 3b + 15  0 (b) a2 – 3b + 20  0 (c) a2 – 3b + 25  0 (d) a2 – 3b + 30  0

 x 3  cos1 a, 0  x  1
Q.16 The range of values of a for which the function f(x) = 
 x, 1  x  3
has the smallest value at x = 1, is
(a) [cos 2, 1] (b) [–1, cos 2] (c) [0, 1] (d) [–1, 1]
1

Q.17 If f (x) =  e|t  x | dt where (0  x  1), then maximum value of f (x) is


0

(a) e – 2 (b) e – 3 (c) e – 1 (d) 2  e 1 


Q.18 In which one of the following intervals Rolle's theorem hold(s) good for
1 1
y = x2 sin + x3 cos .
x 2x
1 2  1 1  1 1 1 3
(a)  ,  (b)  ,  (c)  ,  (d)  , 
    3    2     

2
3
Q.19 Number of critical points of the function f(x) =  x  2  (2x + 1) is equal to
(a) 0 (b) 1 (c) 2 (d) 3


ANSWER KEY

1 2 3 4 5 6 7 8 9 10

A B C A D A D C B C

11 12 13 14 15 16 17 18 19

C C B C A B C B C

Nothing is impossible, the word itself says ‘I’m possible’!


30 APPLICATION OF DERIVATIVES

ADVANCE OBJECTIVE EXERCISE - 5

Paragraph
x2
Consider the function f (x) =
x2  1
Q.1 The interval in which f is increasing is
(a) (–1, 1) (b) (– , – 1)(–1, 0)
(c) (– , ) – {–1, 1} (c) (0, 1)  (1, )

Q.2 If f is defined from R – {–1, 1}  R then f is


(a) injective but not surjective (b) surjective but not injective
(c) injective as well as surjective (d) neither injective nor surjective.

Q.3 f has
(a) local maxima but no local minima (b) local minima but no local maxima
(c) both local maxima and local minima (d) neither local maxima nor local minima.

Paragraph
Consider the cubic f (x) = 8x3 + 4ax2 + 2bx + a where a, b  R.
Q.4 For a = 1 if y = f (x) is strictly increasing  x  R then maximum range of values of b is
 1 1  1 
(a)  , (b)  ,   (c)  ,   (d) (–, )
 3  3  3 

Q.5 For b = 1, if y = f (x) is non monotonic then the sum of all the integral values of
a  [1, 100], is
(a) 4950 (b) 5049 (c) 5050 (d) 5047

Q.6 If the sum of the base 2 logarithms of the roots of the cubic f (x) = 0 is 5 then the value of 'a'
is
(a) – 64 (b) – 8 (c) – 128 (d) – 256
Paragraph

Consider the function f (x) = x30 · (ln x)20 for x > 0

Q.7 If f is continuous at x = 0 then f (0)


(a) is equal to 0
(b) is equal to 2/3
(c) is equal to 1
(d) can not be defined to make f (x) continuous at x=0

CatalyseR Eduventures (India) Pvt. Ltd.


APPLICATION OF DERIVATIVES 31

Q.8 Maximum value of f (x) occurs at


2
(a) x = – (b) x = 1 (c) x = e–2/3 (d) x = e2/3
3
Paragraph

Consider the functions f (x) and g (x) such that


x 1
x3
f (x) =  1  x  g (t ) dt and g (x) = x   f (t ) dt .
2 0 0

Both f (x) and g (x) are defined from R  R.

Q.9 Which one of the following holds good for f (x)?


(a) f (x) is bounded (b) f (x) has exactly one maxima and one minima
(c) f (x) has a maxima but no minima
(d) f (x) has a minima but no maxima

Q.10 Minimum distance between the functions f (x) and g (x) is


4 7 7 8
(a) (b) (c) (d)
3 2 6 2 3 2 3 2

Q.11 The function g (x)


(a) is injective but not surjective (b) cuts the y-axis at – 3/2
(c) cuts the y-axis at 3/2 (d) is neither injective nor surjective

Paragraph

Let f (x) be a real valued continuous function on R defined as f (x) = x2 e– | x |

Q.12 The value of k for which the curve y = kx2 (k > 0) intersect the curve y = e| x | at exactly two
points, is
e2 e2 e2
(a) e2 (b) (c) (d)
2 4 8

Q.13 The area bounded by ordinates at points of local maximum, the curve y = f (x) and x-axis is
equal to
4  e2  5 e2  5 2  e2  5
(a) (b) (c) (d) 4
e2 e e2

Q.14 Range of the function f (x) is


 2  4  4
(a)  0, 2  (b)  0, 2  (c)  0,  (d) (0, e2)
 e   e   e

Nothing is impossible, the word itself says ‘I’m possible’!


32 APPLICATION OF DERIVATIVES

Paragraph

x
Consider the function f(x) = x and g(x) = max. {f (t) : x  t  x + 1}
2
Q.15 Which of the following statement is incorrect?
 1   1 
(a) f(x) is increasing in  ,  and decreasing in  , 
 ln 2   ln 2 
(b) local maximum value of f(x) exists
(c) local minimum value of f(x) does not exists
(d) f(x) changes its concavity at 2 points

Q.16 If f(x) = k has 2 distinct real roots then range of k is equal to


 1  1   1 
(a)  0,  (b)  0,  (c)  ,  (d) (– , 0)
 e  e ln 2   e ln 2 

1
1
ln 2
Q.17 The value of the definite integral  g ( x) dx is equal to
0

1 1 2 1 1
(a) 2
 – (b) 
2 ln 2 2 ln 2 e ln 2 2 2
2 ln 2 2 ln 2
1 1 2 1 1
(c) 2
 + (d) 
2 ln 2 2 ln 2 e ln 2 2 2
2 ln 2 2 ln 2

Paragraph

A curve y = f (x) passing through origin and (2, 4). Through a variable point P (a, b) on the
curve,
lines are drawn parallel to coordinates axes. The ratio of area formed by the curve
y = f (x), x = 0,
y = b to the area formed by the y = f (x), y = 0, x = a is equal to 2 : 1.

Q.18 Equation of line touching both the curves y = f (x) and y2 = 8x is


(a) 2x + y – 1 = 0 (b) 2x + y + 1 = 0 (c) x + 2y + 1 = 0 (d) x + 2y – 1 = 0

Q.19 Pair of tangents are drawn from the point (3, 0) to y = f (x). The area enclosed by these
tangents and y = f (x) is equal to
(a) 9 (b) 18 (c) 15 (d) 27

CatalyseR Eduventures (India) Pvt. Ltd.


APPLICATION OF DERIVATIVES 33

 1
Q.20 AB is the chord of curve y = f (x) passing through  0,  . Locus of point of intersection of
 4
tangents at A and B is
(a) 4y + 1 = 0 (b) 4y – 1 = 0 (c) 4x + 1 = 0 (d) 4x – 1 = 0

ANSWER KEY

1 2 3 4 5 6 7 8 9 10

B D A C B D A C D C

11 12 13 14 15 16 17 18 19 20

B C A B D B A B B A

ADVANCE OBJECTIVE EXERCISE - 6

Q.1 f:RR
Statement-1: f (x) = 12x5 – 15x4 + 20x3–30x2 + 60x + 1 is monotonic and surjective on R.
Statement-2: A continuous function defined on R, if strictly monotonic has its range R.
(a) Statement-1 is true, statement-2 is true and statement-2 is correct explanation for
statement-1.
(b) Statement-1 is true, statement-2 is true and statement-2 is NOT the correct explanation
for statement-1.
(c) Statement-1 is true, statement-2 is false.
(d) Statement-1 is false, statement-2 is true.

2x  2
Q.2 Let f (x) = ln(2 + x) – .
x3
Statement-1: The equation f (x) = 0 has a unique solution in the domain of f (x).
Statement-2: If f (x) is continuous in [a, b] and is strictly monotonic in (a, b) then f has a
unique root in (a, b)
(a) Statement-1 is true, statement-2 is true and statement-2 is correct explanation for
statement-1.
(b) Statement-1 is true, statement-2 is true and statement-2 is NOT the correct explanation
for statement-1.
(c) Statement-1 is true, statement-2 is false.
(d) Statement-1 is false, statement-2 is true.

Nothing is impossible, the word itself says ‘I’m possible’!


34 APPLICATION OF DERIVATIVES

Q.3 Statement-1: The function F (x) = x ln x is increasing in (1/e, )


Statement-2: If f (x) and g (x) both are increasing in (a, b) then f (x) · g(x) must be
increasing in (a, b).
(a) Statement-1 is true, statement-2 is true and statement-2 is correct explanation for
statement-1.
(b) Statement-1 is true, statement-2 is true and statement-2 is NOT the correct explanation
for statement-1.
(c) Statement-1 is true, statement-2 is false.
(d) Statement-1 is false, statement-2 is true.

x7 x6 x5 x4 x3 x 2
Q.4 Consider the polynomial function f (x) =      x
7 6 5 4 3 2
Statement-1: The equation f (x) = 0 can not have two or more roots.
Statement-2: Rolles theorem is not applicable for y = f (x) on any interval [a, b]
where a,b R
(a) Statement-1 is true, statement-2 is true and statement-2 is correct explanation for
statement-1.
(b) Statement-1 is true, statement-2 is true and statement-2 is NOT the correct explanation
for statement-1.
(c) Statement-1 is true, statement-2 is false.
(d) Statement-1 is false, statement-2 is true.

2x if 0  x  1
 2
Q.5 A function y = f (x) is defined on [0, 4] as f (x) =  ( x  2) if 1  x  3
 1 if 3  x  4
For the function y = f (x)
Statement-1: All the three conditions of Rolles Theorem are violated on [0, 4] but still f '
(x) vanishes at a point in (0, 4).
Statement-2: The conditions for Rolles Theorem are sufficient but not necessary.
(a) Statement-1 is true, statement-2 is true and statement-2 is correct explanation for
statement-1.
(b) Statement-1 is true, statement-2 is true and statement-2 is NOT the correct explanation
for statement-1.
(c) Statement-1 is true, statement-2 is false.
(d) Statement-1 is false, statement-2 is true.

CatalyseR Eduventures (India) Pvt. Ltd.


APPLICATION OF DERIVATIVES 35

Q.6 Statement-1: f (x) = loge + x( + x) is strictly increasing for all x > 0.
Statement-2:  + x > e + x  x > 0
(a) Statement-1 is true, statement-2 is true and statement-2 is correct explanation for
statement-1.
(b) Statement-1 is true, statement-2 is true and statement-2 is NOT the correct explanation
for statement-1.
(c) Statement-1 is true, statement-2 is false. (d) Statement-1 is false, statement-2 is true.

Q.7 Consider the graph of the function f (x) = x + | x|


Statement–1: The graph of y = f (x) has only one critical point
Statement-2: f '(x) vanishes only at one point
(a) Statement-1 is true, statement-2 is true and statement-2 is correct explanation for
statement-1.
(b) Statement-1 is true, statement-2 is true and statement-2 is NOT a correct explanation for
statement-1.
(c) Statement-1 is true, statement-2 is false. (d) Statement-1 is false, statement-2 is true.
1
Q.8 Statement-1: The function f (x) = ln x is increasing in (0, 10) and g (x) = is decreasing in
x
(0,10)
Statement-2: If a differentiable function increases in the interval (a, b) then its derivative
function decreases in the interval (a, b).
(a) Statement-1 is true, statement-2 is true and statement-2 is correct explanation for
statement-1.
(b) Statement-1 is true, statement-2 is true and statement-2 is NOT the correct explanation
for statement-1.
(c) Statement-1 is true, statement-2 is false. (d) Statement-1 is false, statement-2 is true.

Q.9 Statement-1: Suppose f (x) = 2x + 1 and g (x) = 4–x + 2–x. The equation f(x) = g(x) has
exactly one root.
Statement-2: If f (x) and g (x) are two differentiable functions defined for all x  R and if
f (x) is strictly increasing and g (x) is strictly decreasing for every x  R then
the equation f (x) = g(x) must have exactly one root.
(a) Statement-1 is true, statement-2 is true and statement-2 is correct explanation for
statement-1.
(b) Statement-1 is true, statement-2 is true and statement-2 is NOT the correct explanation
for statement-1.
(c) Statement-1 is true, statement-2 is false. (d) Statement-1 is false, statement-2 is true.

Nothing is impossible, the word itself says ‘I’m possible’!


36 APPLICATION OF DERIVATIVES

Q.10 Statement 1: Let f(x) = Max.  x 2 , (1  x) 2 , 2 x (1  x)  where 0  x  1. If Rolle's theorem is


4
applicable for f(x) on [, ] then  +  +  = , where   (, ) such
3
that f '() = 0.
Statement 2: If a real-valued function g(x) defined on [a,b] satisfies hypothesis of Rolle's
theorem on [a, b] then g'(c) = 0 for some c  (a, b).
(a) Statement-1 is true, statement-2 is true and statement-2 is correct explanation for
statement-1.
(b) Statement-1 is true, statement-2 is true and statement-2 is NOT the correct explanation
for statement-1.
(c) Statement-1 is true, statement-2 is false. (d) Statement-1 is false, statement-2 is true.
x

Q.11 The function f(x) =  1  t 4 dt is such that :


0

(a) it is defined on the interval [ 1, 1] (b) it is an increasing function


(c) it is an odd function (d) the point (0, 0) is the point of inflection

Q.12 Let f(x) = (x2  1)n (x2 + x + 1) then f(x) has local extremum at x = 1 when
(a) n = 2 (b) n = 3 (c) 4 (d) n=6

Q.13 If f(x) = a
a x 
sgn x
; g(x) = a  
a x sgn x 
for a > 0, a  1 and x  R, where { } & [] denote
the fractional part and integral part functions respectively, then which of the following
statements can hold good for the function h(x), where
(ln a) h(x) = (ln f(x) + ln g(x)).
(a) ‘h’ is even and increasing (b) ‘h’ is odd and decreasing
(c) ‘h’ is even and decreasing (d) ‘h’ is odd and increasing.

Q.14 On which of the following intervals, the function x100 + sin x  1 is strictly increasing.
(a) ( 1, 1) (b) (0, 1) (c) (/2, ) (d) (0, /2)


Q.15 Equation of a tangent to the curve y cot x = y3 tan x at the point where the abscissa is is
4
(a) 4x + 2y =  + 2 (b) 4x  2y =  + 2 (c) x = 0 (d) y=0

Q.16 If the side of a triangle vary slightly in such a way that its circum radius remains constant,
da db dc
then,   is equal to :
cos A cos B cos C
(a) 6 R (b) 2 R (c) 0 (d) 2R(dA + dB + dC)

CatalyseR Eduventures (India) Pvt. Ltd.


APPLICATION OF DERIVATIVES 37

ANSWER KEY

1 2 3 4 5 6 7 8 9 10

C C C A A D D C C D

11 12 13 14 15 16

ABCD ACD D BCD ABD CD

ADVANCE OBJECTIVE EXERCISE - 7


x

Q.1 An extremum value of y = 


0
(t  1) (t  2) dt is :

(a) 5/6 (b) 2/3 (c) 1 (d) 2


x2 if x  1

Q.2 The function f (x) =  x2 if  1  x  1

( x  2) 2 if x  1
(a) is continuous for all x  R
(b) is continuous but not differentiable  x  R
(c) is such that f ' (x) change its sign exactly twice
(d) has two local maxima and two local minima.

Q.3 Equation of a line which is tangent to both the curves y = x2 + 1 and y = – x2 is


1 1
(a) y = 2x  (b) y = 2x 
2 2
1 1
(c) y = – 2x  (d) y = – 2x 
2 2

Q. 4 A function f is defined by f (x) =  cos t cos( x  t )dt , 0  x  2 then which of the following
0

hold(s) good?
(a) f (x) is continuous but not differentiable in (0, 2)
(b) Maximum value of f is 
(c) There exists atleast one c  (0, 2) s.t. f ' (c) = 0.

(d) Minimum value of f is – .
2

Nothing is impossible, the word itself says ‘I’m possible’!


38 APPLICATION OF DERIVATIVES

Q. 5 Which of the following inequalities always hold good in (0, 1)


x2
(a) x > tan–1x (b) cos x < 1 –
2
x2

(c) 1 + x ln x  1  x 2  > 1  x2 (d) x –
2
< ln(1 + x)

x
Q. 6 The function f(x) =  t (et  1) (t  1) (t  2)3 (t  3)5 dt has a local minimum at x =
1

(a) 0 (b) 1 (c) 2 (d) 3

Q. 7 If f (x) is continuous and derivable  x  R and f ' (c) = 0 for exactly 2 real values of 'c'
then the number of real and distinct values of 'd' for which f (d) = 0 can be
(a) 1 (b) 2 (c) 3 (d) 4

Q. 8 If Lim f ( x )  Lim[ f ( x )] (a is a finite quantity), where [·] denotes greatest integer function
x a xa

and f (x) is a non constant continuous function, then


(a) Lim f(x) is an integer. (b) Lim f(x) need not be an integer.
x a x a

(c) f(x) has a local minimum at x = a. (d) f(x) has a local maximum at x = a.

Q. 9 Let y = f (x) be a function such that f ' (x) = x3 and the line x + y = 0 is tangent to the graph
of f (x) then which of the following alternative(s) is/are correct?
3
(a) f (0) = – (b) f (1) = 1 (c) f (–1) = 1 (d) f(3) = 21
4

Q.10 Which of the following functions fail to satisfy the condition of Rolle's theorem on the
interval [– 1, 1],
 tan x
 , x0
(a) f (x) = | x |[x ] ( b) f ( x )   x
 0, x0
(c) f (x) = { x }+ { – x } (d) f (x) = | x | – | sin x |
Note: where [x] denotes the greatest integer less or equal to x and {x} denotes the fractional
part of x respectively.

Q.11 Let f be real-valued function on R defined as f(x) = x4 (1 – x)2 , then which of the
following statement(s) is(are) correct ?
(a) f '(c) = 0 for some c  (0, 1). (b) f "(x) vanishes exactly twice in R.
 2
(c) f (x) is an even function. (d) Monotonic increasing in  0,   1,   .
 3

CatalyseR Eduventures (India) Pvt. Ltd.


APPLICATION OF DERIVATIVES 39

 
Q.12 Let f (x) = 1 + x ln x  x 2  1 – 1  x 2 and h (x) = f (x) – f 2(x) + f 3(x). Then

(a) h (x) is increasing in (0, ). (b) h (x) is decreasing in (–, 0).
(c) h (x) is increasing on R. (d) h (x) in decreasing on R.

Q.13 Let f (x) = a(x – x1)(x – x2) where a, x1 and x2 are real numbers such that a  0 and x1 +
x2 = 0.
Then which of the following statements is/are always correct?
(a) f (x) is increasing in (– , 0) and decreasing in (0, )
(b) f (x) is decreasing in (– , 0) and inecreasing in (0, )
(c) f (x) is non monotonic on R
(d) f (x) has an extremum point

Q.14 Let f : [0, 2]  R be continuous in [0, 2] and differentiable in (0, 2) and


f (0) = 0, f (1) = 1, f (2) = 1. Which of the following statements (is/are) correct?
1
(a) There exist some c1  (0, 1) such that f (c1) = .
10
(b) There exist some c2  (0, 1) such that f ' (c2) = 1.
(c) There exist some c3  (1, 2) such that f ' (c3) = 0.
1
(d) There exist some c4  (0, 2) such that f ' (c4) = .
2
1

Q.15 Let f be a continuous function on R and satisfies f (x) = ex +  e x f (t ) dt ,


0

then which of the following is(are) correct?


(a) f (0) < 0 (b) f (x) is decreasing function on R
1

(c) f (x) is an increasing function on R (d)  f ( x) dx > 0


0

Q.16 Let S be the set of all x such that x4 – 10x2 + 9  0. Find the maximum value of
f (x) = x3 – 3x on S.

1
Q.17 Find sum of all possible integral value(s) of 'p' for which the equation x  3 = p – 3
x
has exactly two distinct solutions.

Nothing is impossible, the word itself says ‘I’m possible’!


40 APPLICATION OF DERIVATIVES

Q.18 For constant number 'a', consider the function f (x) = ax + cos 2x + sin x + cos x on R
(the set of real numbers) such that f (u) < f (v) for u < v. If the range of 'a' for any real
numbers
m 
u, v is  ,   , then find the minimum value of (m + n).
n 

Q.19 Let P be the point on the curve 4x2 + 2 y2 = 42, 0 < 2 < 8 whose distance from
Q(0, – 2) is greatest. If R is the reflection of P in the x-axis then find the least distance of
R from the line 3x – 4y + 7 = 0.

Q.20 Let f : [0, )  R be a continuous, strictly increasing function such that f 3(x) =
x
2 1
t f
0
(t ) dt . If a normal is drawn to the curve y = f (x) with gradient
2
, then find the

intercept made by it on the y-axis.



ANSWER KEY

1 2 3 4 5 6 7 8 9 10

AB ABD AC CD ACD BD ABC AC BCD ABC

11 12 13 14 15 16 17 18 19 20

AD AB CD ABCD AB 18 21 25 3 9

CatalyseR Eduventures (India) Pvt. Ltd.


APPLICATION OF DERIVATIVES 41

1.8. ADVANCE SUBJECTIVE EXERCISE

ADVANCE SUBJECTIVE EXERCISE – 1 (TANGENT & NORMAL)

Q.1 Find the equation of the normal to the curve y = (1 + x)y + sin1 (sin2 x) at x = 0.
Q.2 Find the equations of the tangents drawn to the curve y2 – 2x3 – 4y + 8 = 0 from the point
(1, 2).

Q.3 Find the point of intersection of the tangents drawn to the curve x2 y = 1 – y at the points
where it is intersected by the curve xy = 1 – y.
Q.4 Find all the lines that pass through the point (1, 1) and are tangent to the curve represented
parametrically as x = 2t – t2 and y = t + t2.

7
Q.5 The tangent to y = ax2 + bx + at (1, 2) is parallel to the normal at the point (–2, 2) on the
2
curve y = x2 + 6x + 10. Find the value of a and b.
Q.6 A straight line is drawn through the origin and parallel to the tangent to a curve

x  a2  y2  a  a2  y2 
=   lnat an arbitary point M. Show that the locus of the point P
a  y 
 
of intersection of the straight line through the origin & the straight line parallel to the x-axis
& passing through the point M is x2 + y2 = a2.

41x3
Q.7 A line is tangent to the curve f (x) = at the point P in the first quadrant, and has a slope
3
of 2009. This line intersects the y-axis at (0, b). Find the value of 'b'.
Q.8 A function is defined parametrically by the equations
1 1
 2t  t 2 sin if t  0  sin t 2 if t  0
f(t) = x =  t and g(t) = y =  t
 0 if t  0  o if t  0
Find the equation of the tangent and normal at the point for t = 0 if exist.

Q.9 Find all the tangents to the curve y = cos (x + y),  2  x  2, that are parallel to the line
x + 2y = 0.

Q.10 Prove that the segment of the normal to the curve x = 2a sin t + a sin t cos2t ; y =  a cos3t
contained between the co-ordinate axes is equal to 2a.

Nothing is impossible, the word itself says ‘I’m possible’!


42 APPLICATION OF DERIVATIVES

Q.11 Show that the normals to the curve x = a (cos t + t sin t) ; y = a (sin t  t cos t) are tangent
lines to the circle x2 + y2 = a2.

1
Q.12 The chord of the parabola y =  a2x2 + 5ax  4 touches the curve y = at the point x = 2
1 x
and is bisected by that point. Find 'a'.

Q.13 If the tangent at the point (x1, y1) to the curve x3 + y3 = a3 (a  0) meets the curve again in
x2 y2
(x2, y2) then show that  =  1.
x1 y1

1
Q.14 Determine a differentiable function y = f (x) which satisfies f ' (x) = [f(x)]2 and f (0) = – .
2
Find also the equation of the tangent at the point where the curve crosses the y-axis.
Q.15 Tangent at a point P1 [other than (0 , 0)] on the curve y = x3 meets the curve again at P2. The
tangent at P2 meets the curve at P3 & so on. Show that the abscissae of P1, P2, P3, ......... Pn,
area ( P1 P2 P3 )
form a GP. Also find the ratio .
area ( P2 P3 P4 )

Q.16 The curve y = ax3 + bx2 + cx + 5 , touches the x - axis at P ( 2 , 0) & cuts the y-axis at a point
Q where its gradient is 3. Find a , b , c.

Q.17 The tangent at a variable point P of the curve y = x2  x3 meets it again at Q. Show that the
locus of the middle point of PQ is y = 1  9x + 28x2  28x3.

Q.18 Show that the distance from the origin of the normal at any point of the curve
     
x = a e  sin  2 cos  & y = a e  cos  2 sin  is twice the distance of the tangent at
 2 2  2 2
the point from the origin.

Q.19 Show that the condition that the curves x2/3 + y2/3 = c2/3 & (x2/a2) + (y2/b2) = 1 may touch if
c = a + b.
Q.20 The graph of a certain function f contains the point (0, 2) and has the property that for each
number 'p' the line tangent to y = f (x) at  p , f ( p )  intersect the x-axis at p + 2. Find f (x).

Q.21 A curve is given by the equations x = at 2 & y = at3. A variable pair of perpendicular lines
through the origin 'O' meet the curve at P & Q. Show that the locus of the point of
intersection of the tangents at P & Q is 4y2 = 3ax  a2.

CatalyseR Eduventures (India) Pvt. Ltd.


APPLICATION OF DERIVATIVES 43

Q.22 A and B are points of the parabola y = x2. The tangents at A and B meet at C. The median of
the triangle ABC from C has length 'm' units. Find the area of the triangle in terms of 'm'.

Q.23 (a) Find the value of n so that the subnormal at any point on the curve xyn = an + 1 may be
constant.
(b) Show that in the curve y = a. ln (x2  a2), sum of the length of tangent & subtangent
varies as the product of the coordinates of the point of contact.

x2 y2 x2 y2
Q.24 (a) Show that the curves  = 1 &  = 1 intersect
a2  K1 b2  K1 a 2  K2 b2  K2
orthogonally.
(b) If the two curves C1 : x = y2 and C2 : xy = k cut at right angles find the value of k.

Q.25 Show that the angle between the tangent at any point 'A' of the curve
y
ln (x2 + y2) = C tan–1 and the line joining A to the origin is independent of the position of
x
A on the curve.


ANSWER KEY
Q.1 x + y – 1 = 0 Q.2 2 3x  y= 2  
3  1 or 2 3 x+y=2  3 1
Q.3 (0, 1) Q.4 x = 1 when t = 1, m  ; 5x – 4y = 1 if t  1, t = 1/3
5 82·7 3
Q.5 a = 1, b = Q.7 – Q.8 T : x – 2y = 0 ; N : 2x + y = 0
2 3
Q.9 x + 2 y = /2 & x + 2 y =  3 /2 Q.12 a = 1
1
Q.14 – ; x – 4y = 2 Q.15 1/16 Q.16 a =  1/2 ; b =  3/4 ; c = 3
x2
m m
Q.20 2e–x/2 Q.22 Q.23 (a) n =  2
2
1 2
Q.24 (b) ± Q.25  = tan–1
2 2 C

Nothing is impossible, the word itself says ‘I’m possible’!


44 APPLICATION OF DERIVATIVES

ADVANCE SUBJECTIVE EXERCISE – 2 (RATE MEASURE AND APPROXIMATIONS)

Q.1 Water is being poured on to a cylindrical vessel at the rate of 1 m3/min. If the vessel has a
circular base of radius 3 m, find the rate at which the level of water is rising in the vessel.

Q.2 A man 1.5 m tall walks away from a lamp post 4.5 m high at the rate of 4 km/hr.
(i) how fast is the farther end of the shadow moving on the pavement ?
(ii) how fast is his shadow lengthening ?

Q.3 A particle moves along the curve 6 y = x3 + 2. Find the points on the curve at which the y
coordinate is changing 8 times as fast as the x coordinate.

Q.4 An inverted cone has a depth of 10 cm & a base of radius 5 cm. Water is poured into it at the
rate of 1.5 cm3/min. Find the rate at which level of water in the cone is rising, when the depth
of water is 4 cm.
Q.5 A water tank has the shape of a right circular cone with its vertex down. Its altitude is 10 cm
and the radius of the base is 15 cm. Water leaks out of the bottom at a constant rate of 1cu.
cm/sec. Water is poured into the tank at a constant rate of C cu. cm/sec. Compute C so that
the water level will be rising at the rate of 4 cm/sec at the instant when the water is 2 cm
deep.

Q.6 Sand is pouring from a pipe at the rate of 12 cc/sec. The falling sand forms a cone on the
ground in such a way that the height of the cone is always 1/6th of the radius of the base.
How fast is the height of the sand cone increasing when the height is 4 cm.

Q.7 An open Can of oil is accidently dropped into a lake; assume the oil spreads over the surface
as a circular disc of uniform thickness whose radius increases steadily at the rate of 10
cm/sec. At the moment when the radius is 1 meter, the thickness of the oil slick is decreasing
at the rate of 4 mm/sec, how fast is it decreasing when the radius is 2 meters.

Q.8 Water is dripping out from a conical funnel of semi vertical angle /4, at the uniform rate of
2 cm3/sec through a tiny hole at the vertex at the bottom. When the slant height of the water
is 4 cm, find the rate of decrease of the slant height of the water.

Q.9 An air force plane is ascending vertically at the rate of 100 km/h. If the radius of the earth is
R Km, how fast the area of the earth, visible from the plane increasing at 3min after it started
2 R 2 h
ascending. Take visible area A = Where h is the height of the plane in kms above the
Rh
earth.

CatalyseR Eduventures (India) Pvt. Ltd.


APPLICATION OF DERIVATIVES 45

Q.10 A variable  ABC in the xy plane has its orthocentre at vertex 'B' , a fixed vertex 'A' at the
7 x2
origin and the third vertex 'C' restricted to lie on the parabola y = 1 + . The point B starts
36
at the point (0, 1) at time t = 0 and moves upward along the y axis at a constant velocity of 2
7
cm/sec. How fast is the area of the triangle increasing when t = sec.
2
Q.11 A circular ink blot grows at the rate of 2 cm2 per second. Find the rate at which the radius is
6 22
increasing after 2 seconds. Use  = .
11 7
Q.12 Water is flowing out at the rate of 6 m3/min from a reservoir shaped like a hemispherical
bowl of radius R = 13 m. The volume of water in the hemispherical bowl is given by V =
 2
· y (3R  y ) when the water is y meter deep. Find
3
(a) At what rate is the water level changing when the water is 8 m deep.
(b) At what rate is the radius of the water surface changing when the water is 8 m deep.
Q.13 If in a triangle ABC, the side 'c' and the angle 'C' remain constant, while the remaining
da db
elements are changed slightly, show that  = 0.
cos A cos B
Q.14 At time t > 0, the volume of a sphere is increasing at a rate proportional to the reciprocal of
its radius. At t = 0, the radius of the sphere is 1 unit and at t = 15 the radius is 2 units.
(a) Find the radius of the sphere as a function of time t.
(b) At what time t will the volume of the sphere be 27 times its volume at t = 0.

Q.15 (i) Use differentials to a approximate the values of ; (a) 36.6 and (b) 3 26 .
(ii) If the radius of a sphere is measured as 9 cm with an error of 0.03 cm, then find the
approximate error in calculating its volume.

ANSWER KEY
Q.1 1/9  m/min Q.2 (i) 6 km/h ; (ii) 2 km/hr Q.3 (4 , 11) & ( 4,  31/3)
Q.4 3/8  cm/min Q.5 1 + 36  cu. cm/sec Q.6 1/48  cm/s
2
Q.7 0.05 cm/sec Q.8 cm/s Q.9 200 r3 / (r + 5)² km² / h
4
66 1
Q.10 Q.11 cm/sec.
7 4
1 5
Q.12 (a) – m/min., (b) – m/min. Q.14 (a) r = (1 + t)1/4, (b) t = 80
24  288 
80
Q.15 (i) (a) 6.05, (b) ; (ii) 9.72 cm3
27

Nothing is impossible, the word itself says ‘I’m possible’!


46 APPLICATION OF DERIVATIVES

ADVANCE SUBJECTIVE EXERCISE – 3 (MONOTONICITY)

Q.1 Find the intervals of monotonocity for the following functions & represent your solution set
on the number line.
2
(a) f(x) = 2. e x  4 x (b) f(x) = ex/x (c) f(x) = x2 ex (d) f (x) = 2x2 – ln | x |
Also plot the graphs in each case & state their range.

Q.2 Let f (x) = 1 – x – x3. Find all real values of x satisfying the inequality,
1 – f (x) – f 3(x) > f (1 – 5x)

Q.3 Find the intervals of monotonocity of the functions in [0, 2]


(a) f (x) = sin x – cos x in x [0 , 2 ] (b) g (x) = 2 sinx + cos 2x in (0  x  2 ).
4 sin x  2 x  x cos x
(c) f (x) =
2  cos x

Q.4 Let f (x) be a increasing function defined on (0, ). If f (2a2 + a + 1) > f (3a2 – 4a + 1). Find
the range of a.

max{ f (t ) : 0  t  x} , 0  x  1

Q.5 Let f (x) = x3  x2 + x + 1 and g(x) = 
 3 x ,1  x  2
Discuss the conti. & differentiability of g(x) in the interval (0,2).

Q.6 Find the set of all values of the parameter 'a' for which the function,
f(x) = sin 2x – 8(a + 1)sin x + (4a2 + 8a – 14)x increases for all x  R and has no critical
points for all x  R.

Q.7 Find the greatest & the least values of the following functions in the given interval if they
exist.
x  1 
(a) f (x) = sin1  ln x in  , 3 
x2  1  3 
(b) f (x) = 12x4/3 – 6x1/3, x  [–1, 1]
(c) y = x5 – 5x4 + 5x3 + 1 in [ 1, 2]

1
Q.8 Find the values of 'a' for which the function f(x) = sin x  a sin2x  sin3x + 2ax increases
3
throughout the number line.

CatalyseR Eduventures (India) Pvt. Ltd.


APPLICATION OF DERIVATIVES 47

 a2 1  3 2
Q.9 If f(x) =   x + (a - 1) x + 2x + 1 is monotonic increasing for every x  R then find the
 3 
range of values of ‘a’.

Q.10 Find the set of values of 'a' for which the function,
 21  4 a  a 2  3
f(x) = 1   x + 5x + 7 is increasing at every point of its domain.
 a 1 
 

dg
Q.11 Let a + b = 4 , where a < 2 and let g (x) be a differentiable function. If > 0 for all x,
dx
a b

prove that  g ( x) dx   g ( x) dx increases as (b  a) increases.


0 0

x x 1
 1  1
Q.12 Let f (x) = 1   and g (x) = 1   , both f and g being defined for x > 0, then prove
 x  x
that f (x) is increasing and g (x) is decreasing.
x2
1  t 1 
Q.13 Find the value of x > 1 for which the function F (x) =  t ln  dt is increasing and
x
32 
decreasing.

Q.14 Find all the values of the parameter 'a' for which the function ;
f(x) = 8ax  a sin 6x  7x  sin 5x increases & has no critical points for all x  R.

Q.15 If f (x) = 2ex – ae–x + (2a + 1)x  3 monotonically increases for every x  R then find the
range of values of ‘a’.

Q.16 Prove that, x2 – 1 > 2x ln x > 4(x – 1) – 2 ln x for x > 1.

 3 
Q.17 Prove that tan2x + 6 ln secx + 2cos x + 4 > 6 sec x for x   , 2  .
 2 

Q.18 Find the set of values of x for which the inequality ln (1 + x) > x/(1 + x) is valid.

Q.19 If b > a, find the minimum value of (x  a)3+ (x  b)3, x  R.
x2
Q.20 Suppose that the function f (x) = log c is defined for all x in the interval [a, b], is
x2
monotonic decreasing. Find the value of 'c' for which there exists 'a' and 'b' (b > a > 2) such
that the range of the function is [logcc(b–1), logcc(a–1)].


Nothing is impossible, the word itself says ‘I’m possible’!


48 APPLICATION OF DERIVATIVES

ANSWER KEY
Q.1 (a) I in (2 , ) & D in ( , 2) (b) I in (1 , ) & D in (  , 0)  (0 , 1)
(c) I in (0, 2) & D in ( , )  (2 , )
1 1 1 1
(d) I for x > or  < x < 0 & D for x <  or 0 < x <
2 2 2 2
Q.2 (–2, 0)  (2, )
Q.3 (a) I in [0, 3/4)  (7/4 , 2 ] & D in (3/4 , 7 /4)
(b) I in [0 , /6)  (/2 , 5/6)  (3/2 , 2 ] & D in (/6 , /2)  (5/6, 3 /2)]
(c) I in [0, /2)  (3/2, 2] and D in (/2, 3/2)
Q.4 (0, 1/3)  (1, 5) Q.5 continuous but not diff. at x = 1
Q.6 
a <  2 5  or a > 5

Q.7 (a) (/6)+(1/2)ln 3, (/3) – (1/2)ln 3


(b) Maximum at x = 1 and f (–1) = 18; Minimum at x = 1/8 and f (1/8) = – 9/4
(c) 2 & 10
Q.8 [1, ) Q.9 a  (– , – 3]  [1 , )
Q.10 [ 7,  1)  [2, 3] Q.13  in (3, ) and  in (1, 3)
Q.14 (6, ) Q.15 a  0
Q.18 ( 1, 0)  (, ) Q.19 (b  a)3/4
1
Q.20 0<c<
9

CatalyseR Eduventures (India) Pvt. Ltd.


APPLICATION OF DERIVATIVES 49

ADVANCE SUBJECTIVE EXERCISE – 4 (ROLLE’S & LMVT

Q.1 Verify Rolles throrem for f(x) = (x  a)m (x  b)n on [a, b] ; m, n being positive integer.

Q.2 Let f (x) = 4x3  3x2  2x + 1, use Rolle's theorem to prove that there exist c, 0< c <1 such
that f(c) = 0.

 
Q.3 Using LMVT prove that : (a) tan x > x in  0,  , (b) sin x < x for x > 0
 2

Q.4 Let f be continuous on [a, b] and assume the second derivative f " exists on (a, b). Suppose
that the graph of f and the line segment joining the point  a , f ( a )  and  b, f (b )  intersect at
a point  x0 , f ( x0 )  where a < x0 < b. Show that there exists a point c  (a, b) such that
f "(c) = 0.

Q.5 Prove that if f is differentiable on [a, b] and if f (a) = f (b) = 0 then for any real  there is an
x  (a, b) such that  f (x) + f ' (x) = 0.

3 x0
 2
Q.6 For what value of a, m and b does the function f (x) =   x  3 x  a 0  x  1
 mx  b 1 x  2
satisfy the hypothesis of the mean value theorem for the interval [0, 2].

Q.7 Assume that f is continuous on [a, b], a > 0 and differentiable on an open interval (a, b).
f (a) f (b)
Show that if = , then there exist x0  (a, b) such that x0 f '(x0) = f (x0).
a b

Q.8 Let f, g be differentiable on R and suppose that f (0) = g (0) and f ' (x)  g ' (x) for all x  0.
Show that f (x)  g (x) for all x  0.

Q.9 Let f be continuous on [a, b] and differentiable on (a, b). If f (a) = a and f (b) = b, show that
there exist distinct c1, c2 in (a, b) such that f ' (c1) + f '(c2) = 2.

Q.10 Let f defined on [0, 1] be a twice differentiable function such that, | f " (x) |  1 for all
x  [0, 1] If f (0) = f (1), then show that, | f ' (x) | < 1 for all x  [0, 1]

Q.11 f (x) and g (x) are differentiable functions for 0  x  2 such that f (0) = 5, g (0) = 0, f (2) = 8,
g (2) = 1. Show that there exists a number c satisfying 0 < c < 2 and f ' (c) = 3 g' (c) .

Nothing is impossible, the word itself says ‘I’m possible’!


50 APPLICATION OF DERIVATIVES

Q.12 If f, ,  are continuous in [a, b] and derivable in ]a, b[then show that there is a value of c
f (a ) f (b ) f  (c )
lying between a & b such that,  ( a )  (b )   (c ) = 0
 ( a )  ( b)   ( c )

Q.13 Show that exactly two real values of x satisfy the equation x2 = x sinx + cos x.

Q.14 Let a > 0 and f be continuous in [–a, a]. Suppose that f ' (x) exists and f ' (x)  1 for all x 
(–a, a). If f (a) = a and f (– a) = – a, show that f (0) = 0.

Q.15 Prove the inequality ex > (1 + x) using LMVT for all x  R0 and use it to determine which of
the two numbers e and e is greater.

ANSWER KEY
mb  na
Q.1 c= which lies between a & b Q.6 a = 3, b = 4 and m = 1
mn

ADVANCE SUBJECTIVE EXERCISE – 5 (MAXIMA & MINIMA)

Q.1 A cubic f(x) vanishes at x = 2 & has relative minimum/maximum at x = 1 and x =


1
14
1/3. If  f ( x) dx = , find the cubic f (x).
1
3

Q.2 Investigate for maxima & minima for the function,


x

f (x) =  1
[2 (t  1) (t  2)3 + 3 (t  1)2 (t  2)2] dt

Q.3 Find the greatest & least value for the function ;
(a) y = x + sin 2x , 0  x  2  (b) y = 2 cos 2x  cos 4x , 0  x  

Q.4 Suppose f(x) is a function satisfying the following conditions :


(i) f(0) = 2, f(1) = 1
(ii) f has a minimum value at x = and
2 ax 2ax  1 2 ax  b  1
(iii) for all x, f  (x) = b b 1 1
2( ax  b ) 2 ax  2b  1 2 ax  b
Where a, b are some constants. Determine the constants a, b & the function f(x).

CatalyseR Eduventures (India) Pvt. Ltd.


APPLICATION OF DERIVATIVES 51

Q.5 Suppose f(x) is real valued polynomial function of degree 6 satisfying the following
conditions ;
(a) f has minimum value at x = 0 and 2
(b) f has maximum value at x = 1
f ( x)
x 1 0
(c) for all x, Limit ln 0 1 1 = 2. Determine f (x).
x0
x
1 0 1
x

Q.6 Find the maximum perimeter of a triangle on a given base ‘a’ and having the given vertical
angle.

Q.7 The length of three sides of a trapezium are equal, each being 10 cms. Find the maximum
area of such a trapezium.

Q.8 The plan view of a swimming pool consists of a semicircle of radius r attached to a rectangle
of length '2r' and width 's'. If the surface area A of the pool is fixed, for what value of 'r' and
's' the perimeter 'P' of the pool is minimum.

Q.9 For a given curved surface of a right circular cone when the volume is maximum, prove that
1
the semi vertical angle is sin1 .
3

6
Q.10 Of all the lines tangent to the graph of the curve y = 2
, find the equations of the tangent
x 3
lines of minimum and maximum slope.

Q.11 A statue 4 metres high sits on a column 5.6 metres high. How far from the column must a
man, whose eye level is 1.6 metres from the ground, stand in order to have the most
favourable view of statue.

Q.12 By the post office regulations, the combined length & girth of a parcel must not exceed 3
metre. Find the volume of the biggest cylindrical (right circular) packet that can be sent by
the parcel post.

Q.13 A running track of 440 ft. is to be laid out enclosing a football field, the shape of which is a
rectangle with semi circle at each end. If the area of the rectangular portion is to be
maximum, find the length of its sides. Use :  .

Nothing is impossible, the word itself says ‘I’m possible’!


52 APPLICATION OF DERIVATIVES

Q.14 A window of fixed perimeter (including the base of the arch) is in the form of a rectangle
surmounted by a semicircle. The semicircular portion is fitted with coloured glass while the
rectangular part is fitted with clean glass. The clear glass transmits three times as much light
per square meter as the coloured glass does. What is the ratio of the sides of the rectangle so
that the window transmits the maximum light?

Q.15 A closed rectangular box with a square base is to be made to contain 1000 cubic feet. The
cost of the material per square foot for the bottom is 15 paise, for the top 25 paise and for the
sides 20 paise. The labour charges for making the box are Rs. 3/-. Find the dimensions of the
box when the cost is minimum.

Q.16 Find the area of the largest rectangle with lower base on the x-axis & upper vertices on the
curve y = 12  x2.

Q.17 A trapezium ABCD is inscribed into a semicircle of radius l so that the base AD of the
trapezium is a diameter and the vertices B & C lie on the circumference. Find the base angle
 of the trapezium ABCD which has the greatest perimeter.

ax  b
Q.18 If y = has a turning value at (2, 1) find a & b and show that the turning value
( x 1) ( x  4)
is a maximum.

Q.19 If r is a real number then find the smallest possible distance from the origin (0, 0) to the
vertex of the parabola whose equation is y = x2 + rx + 1.

Q.20 A sheet of poster has its area 18 m². The margin at the top & bottom are 75 cms and at the
sides
50 cms. What are the dimensions of the poster if the area of the printed space is maximum?

x2 y2
Q.21 A perpendicular is drawn from the centre to a tangent to an ellipse 2 + 2 = 1. Find the
a b
greatest value of the intercept between the point of contact and the foot of the perpendicular.

Q.22 A beam of rectangular cross section must be sawn from a round log of diameter d. What
should the width x and height y of the cross section be for the beam to offer the greatest
resistance (a) to compression; (b) to bending. Assume that the compressive strength of a
beam is proportional to the area of the cross section and the bending strength is proportional
to the product of the width of section by the square of its height.

CatalyseR Eduventures (India) Pvt. Ltd.


APPLICATION OF DERIVATIVES 53

Q.23 What are the dimensions of the rectangular plot of the greatest area which can be laid out
within a triangle of base 36 ft. & altitude 12 ft ? Assume that one side of the rectangle lies on
the base of the triangle.

Q.24 The flower bed is to be in the shape of a circular sector of radius r & central angle . If the
area is fixed & perimeter is minimum, find r and .

Q.25 The circle x2 + y2 = 1 cuts the x-axis at P & Q. Another circle with centre at Q and
varable radius intersects the first circle at R above the x-axis & the line segment PQ at S.
Find the maximum area of the triangle QSR.

ANSWER KEY
Q.1 f (x) = x3 + x2  x + 2 Q.2 max. at x = 1 ; f(1) = 0 , min. at x = 7/5 ; f(7/5) =  108/3125

Q.3 (a) Max at x = 2 , Max value = 2 , Min. at x = 0 , Min value = 0


(b) Max at x = /6 & also at x = 5 /6 and
Max value = 3/2 , Min at x = /2 , Min value =  3
1 5 1 2
Q.4 a= ; b =  ; f(x) = (x  5x + 8)
4 4 4
2 6 12 5  
Q.5 f (x) = x  x + 2x4 Q.6 Pmax = a 1  cos ec  Q.7 75 3 sq. units
3 5  2
2A 2A
Q.9 r= ,s = Q.10 3x + 4y – 9 = 0 ; 3x – 4y + 9 = 0
 4  4

Q.11 4 2m Q.12 1/ cu m


Q.13 110 ' , 70 ' Q.14 6/(6 + ) Q.15 side 10', height 10'
Q.16 32 sq. units Q.17  = 600 Q.18 a = 1, b = 0
3
Q.19 dmin = when r = 2 or – 2
2
Q.20 width 2 3 m, length 3 3 m Q.21 a  b
d d 2
Q.22 (a) x = y = , (b) x = , y= d Q.23 6' × 18'
2 3 3
4
Q.24 r = A ,  = 2 radians Q.25
3 3

Nothing is impossible, the word itself says ‘I’m possible’!


54 APPLICATION OF DERIVATIVES

ADVANCE SUBJECTIVE EXERCISE - 6

3
Q.1 The mass of a cell culture at time t is given by, M (t) =
1  4e  t
(a) Find Lim M (t ) and Lim M (t )
t   t 

dM 1
(b) Show that = M (3  M )
dt 3
(c) Find the maximum rate of growth of M and also the vlaue of t at which occurs.

Q.2 Find the cosine of the angle at the vertex of an isosceles triangle having the greatest area for
the given constant length l of the median drawn to its lateral side.

Q.3 From a fixed point A on the circumference of a circle of radius 'a', let the perpendicular AY
fall on the tangent at a point P on the circle, prove that the greatest area which the APY can
a2
have is 3 3 sq. units.
8

Q.4 Given two points A ( 2 , 0) & B (0 , 4) and a line y = x. Find the co-ordinates of a point M on
this line so that the perimeter of the  AMB is least.

Q.5 A given quantity of metal is to be casted into a half cylinder i.e. with a rectangular base and
semicircular ends. Show that in order that total surface area may be minimum , the ratio of
the height of the cylinder to the diameter of the semi circular ends is /(+ 2).

Q.6 Let ,  be real numbers with 0     and f (x) = x2 – ( + )x +  such that  f ( x) dx
1
=


1. Find the maximum value of  f ( x) dx .
0

Q.7 Show that for each a > 0 the function eax. xa² has a maximum value say F (a) , and that F
(x) has a minimum value, ee/2.

1a
3
Q.8 For a > 0, find the minimum value of the integral  (a  4 x  a 5 x 2 )e ax dx .
0

CatalyseR Eduventures (India) Pvt. Ltd.


APPLICATION OF DERIVATIVES 55

x ln x when x  0

Q.9 Consider the function f (x) = 
 0 for x  0
(a) Find whether f is continuous at x = 0 or not.
(b) Find the minima and maxima if they exist.
(c) Does f ' (0) ? Find Lim f '( x ) .
x 0

(d) Find the inflection points of the graph of y = f (x)..

Q.10 Consider the function y = f (x) = ln (1 + sin x) with – 2  x  2. Find


(a) the zeroes of f (x)
(b) inflection points if any on the graph
(c) local maxima and minima of f (x)
(d) asymptotes of the graph
 2
(e) sketch the graph of f (x) and compute the value of the definite integral  f ( x ) dx .
 2

Q.11 The graph of the derivative f ' of a continuous


function f is shown with f (0) = 0. If
(i) f is monotomic increasing in the interval
[a, b)(c, d)(e, f] and decreasing in
(p, q)(r, s).
(ii) f has a local minima at x = x1 and x = x2.
(iii) f is concave up in (l, m)  (n, t]
(iv) f has inflection point at x = k
(v) number of critical points of y = f (x) is 'w'.
Find the value of (a + b + c + d + e) + (p + q + r + s) + (l + m + n) + (x1 + x2) + (k + w).

Q.12 The graph of the derivative f ' of a continuous


function f is shown with f (0) = 0
(i) On what intervals is f increasing or decreasing?
(ii) At what values of x does f have a local
maximum or minimum?
(iii) On what intervals is f concave upward or downward?
(iv) State the x-coordinate(s) of the point(s) of inflection.
(v) Assuming that f (0) = 0, sketch a graph of f.

Nothing is impossible, the word itself says ‘I’m possible’!


56 APPLICATION OF DERIVATIVES

3 2 5
Q.13 Find the set of value of m for the cubic x3 – x + = log1 4 (m) has 3 distinct solutions.
2 2
 2
Q.14 Find the positive value of k for the value of the definite integral  cos x  kx dx is
0

minimised.

Q.15 A cylinder is obtained by revolving a rectangle about the x  axis , the base of the rectangle
lying on the x  axis and the entire rectangle lying in the region between the curve
x
y= 2
& the x  axis. Find the maximum possible volume of the cylinder.
x 1

Q.16 The value of 'a' for which f (x) = x3 + 3 (a  7)x2 + 3 (a2  9)x  1 have a positive point of
maximum lies in the interval (a1, a2)  (a3, a4). Find the value of a2 + 11a3 + 70a4.

Q.17 What is the radius of the smallest circular disk large enough to cover every acute isosceles
triangle of a given perimeter L?

Q.18 Find the magnitude of the vertex angle ‘’ of an isosceles triangle of the given area ‘A’ such
that the radius ‘r’ of the circle inscribed into the triangle is the maximum.

Q.19 The function f (x) defined for all real numbers x has the following properties
(i) f (0) = 0, f (2) = 2 and f ' (x) = k(2x – x2)e–x for some constant k > 0. Find
(a) the intervals on which f is increasing and decreasing and any local maximum or
minimum values.
(b) the intervals on which the graph f is concave down and concave up.
(c) the function f (x) and plot its graph.

Q.20 Use calculus to prove the inequality, sin x  2x  in 0  x  /2.


Use this inequality to prove that, cos x  1 – x 2  in 0  x  /2.



CatalyseR Eduventures (India) Pvt. Ltd.


APPLICATION OF DERIVATIVES 57

ANSWER KEY
3 6
Q.1 (a) 0, 3, (c) , t = ln 4 Q.2 cos A = 0.8 Q.4 (0 , 0) Q.6
4 108
Q.8 4 when a = 2 Q.9 (a) f is continuous at x = 0 ; (b) – 2/e ;
(c) does not exist, does not exist ; (d)pt.of inflection x = 1
Q.10 (a) x = – 2, – , 0, , 2, (b) no inflection point, (c) maxima at x = /2 and – 3/2 and no
minima, (d) x = 3/2 and x = – /2, (e) –  ln 2
Q.11 74 Q.12 (i) I in (1, 6)  (8, 9) and D in (0, 1)  (6, 8);
(ii) L.Min. at x = 1 and x = 8; L.Max. x = 6
(iii) CU in (0, 2)  (3, 5)  (7, 9) and CD in (2, 3)  (5, 7); (iv) x = 2, 3, 5, 7
 1 1 
(v) Graph is Q.13 m   , 
 32 16 
2 2   
Q.14 k= cos   Q.15 /4
 2 2
Q.16 320 Q.17 L/4 Q.18 /3
Q.19 (a) increasing in (0, 2) and decreasing in (–, 0)  (2, ), local min. value = 0 and local
max. value = 2
(b) concave up for (– , 2 – 2 )  (2 + 2 , ) and concave down in (2 – 2 ), (2 + 2 )
1 2·x 2
(c) f (x) = e ·x
2

Nothing is impossible, the word itself says ‘I’m possible’!


58 APPLICATION OF DERIVATIVES

1.9. JEE (MAIN) CORNER

x 2
1. The function f(x) =  has a local minimum at [2006]
2 x
(a) x = – 2 (b) x = 0 (c) x = 1 (d) x = 2

2. A value of c for which the conclusion of Mean Value Theorem holds for the function
f(x) = logex on the interval [1, 3] is [2007]
1
(a) 2 log3e (b) loge 3 (c) log3e (d) loge 3
2

3. The function f(x) = tan–1 (sin x + cos x) is an increasing function in [2007]


(a) (/4, /2) (b) (–/2, /4) (c) (0/2) (d) (–/2, /2)

4. Suppose the cubic x3 – px + q = 0 has three distinct real roots where p > 0 and q > 0. Then,
which one of the following holds ? [2008]
p p
(a) Minima at and maxima at –
3 3
p p
(b) Minima at – and maxima at
3 3
p p
(c) Minima at both and –
3 3
p p
(d) Maxima at both and –
3 3

5. The shortest distance between the line y – x = 1 and the curve x = y2 is [2009]
3 2 2 3 3 2 3
(a) (b) (c) (d)
8 8 5 4

6. Given P(x) = x4 + ax3 + bx2 + cx + d such that x = 0 is the only real root of P(x) = 0.
If P(–1) < P(1), then in the interval [–1, 1] [2009]
(a) P (–1) is the minimum and P(1) is the maximum of P
(b) P (–1) is not minimum but P(1) is the maximum of P
(c) P (–1) is the minimum and P(1) is not the maximum of P
(d) neither P (–1) is the minimum nor P(1) is the maximum of P

CatalyseR Eduventures (India) Pvt. Ltd.


APPLICATION OF DERIVATIVES 59

4
7. The equation of the tangent to the curve y = x + , that is parallel to the x-axis, is
x2
[2010]
(a) y = 1 (b) y = 2 (c) y = 3 (d) y = 0

k – 2 x , if x  –1
8. Let f : R R be defined by f(x) =  [2010]
 2 x  3 , if x  –1
If f has a local minimum at x = – 1, then a possible value of k is
1
(a) 0 (b) – (c) –1 (d) 1
2

1
9. Let f : R ® R be a continuous function defined by f(x) = [2010]
e  2e – x
x

1 1
Statement -1 :f(c) = , for some c Î R. Statement -2 :0 < f(x) £ , for all x Î R.
3 2 2
(a) Statement -1 is true, Statement-2 is true ; Statement -2 is not a correct explanation for
Statement -1.
(b) Statement-1 is true, Statement-2 is false.
(c) Statement -1 is false, Statement -2 is true.
(d) Statement -1 is true, Statement -2 is true; Statement-2 is a correct explanation for
Statement-1.

10. The shortest distance between line y  x  1 and curve x  y 2 is [2011]


3 2 8 4 3
(a) (b) (c) (d)
8 3 2 3 4

x
 5 
11. For x   0,  , define f (x)   t sin t dt . Then f has [2011]
 2  0

(a) local minimum at  and 2 (b) local minimum at  and local maximum at 2
(c) local maximum at  and local minimum at 2
(d) local maximum at  and 2

12. The curve that passes through the point (2, 3), and has the property that the segment of any
tangent to it lying between the coordinate axes is bisected by the point of contact is given by
[2011 RS]
2 2
6 2 2  x  y
(a) 2 y  3x  0 (b) y  (c) x  y  13 (d)       2
x 2  3

Nothing is impossible, the word itself says ‘I’m possible’!


60 APPLICATION OF DERIVATIVES

 tan x
 , x0
13. Let f be a function defined by – f (x)   x
 1, x0
Statement – 1 : x = 0 is point of minima of f
Statement – 2 :f’(0) = 0. [2011 RS]
(a) Statement – 1 is true, statement -2 is true; statement -2 is a correct explanation for
statement -1.
(b) Statement-1 is ture, statement-2 is true; statement-2 is NOT a correct explanation for
statement- 1.
(c) Statement -1 is true, statement -2 is false. (d) Statement -1 is false, statement -2 is true.

14. A spherical balloon is filled with 4500 cubic meters of helium gas .If a leak in the balloon
causes the gas to escape at the rate of 72 cubic meters per minute, then the rate (in meters
per minute) at which the radius of the balloon decreases 49 minutes after the leakage began is
[2012]
9 7 2 9
(a) (b) (c) (d)
7 9 9 2

15. Let a, b R be such that the function given by f (x)  l n | x |  bx 2  ax, x  0 has extreme
values at x   1 and x  2
Statement -1 : f has local maximum at x   1 and at x = 2.
1 1
Statement -2 : a  and b  [2012]
2 4
(a) Statement-1 is false, Statement-2 is true.
(b) Statement-1 is true, statement-2 is true; statement-2 is a correct explanation for
Statement-1.
(c) Statement-1 is true, statement-2 is true; statement-2 is not a correct explanation for
Statement-1.
(d) Statement-1 is true, statement-2 is false.

16. A line is drawn thrugh the point (1, 2) to meet the coordinate axes at P and Q such that it
forms a triangle OPQ, where O is the origin. If the area of the triangle OPQ is least, then the
slope of the line PQ is : [2012]
1
(a)  (b)  4
4
1
(c)  2 (d) 
2

CatalyseR Eduventures (India) Pvt. Ltd.


APPLICATION OF DERIVATIVES 61

x
17. The intercepets on x-axis made by tangents to the curve, y   | t | dt , x  R , which are
0

parallel to the line y = 2x, are equal to: [2013]


(a)  1 (b)  2 (c)  3 (d)  4

18. If f and g are differentiable functions in [0, 1] satisfying f(0) = 2 = g(1), g(0) = 0 and
f(1) = 6, then for some c  ]0, 1[: [2014]
(a) 2f’(c) = 3g’(c) (b) f’(c) = g’(c) (c) f’(c) = 2g’(c) (d) 2f’(c) = g’(c)

19. The slope of the line touching both the parabolas y 2  4 x and x 2  32 y is : [2014]
3 1 2 1
(a) (b) (c) (d)
2 8 3 2

20. Let the population of rabbits surviving at a time t be governed by the differential equation
dp(t) 1
 p (t)  200 . If p(0)  100 , then p(t) equals: [2014]
dt 2
(a) 300  200 et / 2 (b) 600  500 et / 2 (c) 400  300 e t / 2 (d) 400  300 et / 2

21. The normal to the curve, x 2  2 xy  3 y 2  0 , at 1,1 : [2015]


(a) does not meet the curve again
(b) meets the curve again in the second quadrant
(c) meets the curve again in the third quadrant
(d) meets the curve again in the fourth quadrant

22. A wire of length 2 units is cut into two parts which are bent respectively to form a square of
side = x units and a circle of radius = r units. If the sum of the areas of the square and the
circle so formed is minimum, then : [2016]
(a) 2 x  r (b) 2 x     4  r (c)  4    x   r (d) x  2 r

23. Twenty meters of wire is available for fencing off a flower-bed in the form of a circular
sector. Then the maximum area (in sq. m) of the flower-bed, is : [2017]
(a) 12.5 (b) 10 (c) 25 (d) 30

24. The normal to the curve y  x  2  x  3  x  6 at the point where the curve intersects the y-
axis passes through the point: [2017]
 1 1 1 1 1 1 1 1
(a)   ,   (b)  ,  (c)  ,   (d)  , 
 2 2  2 2  2 3  2 3


Nothing is impossible, the word itself says ‘I’m possible’!


62 APPLICATION OF DERIVATIVES

ANSWER - KEY
1 2 3 4 5 6 7 8 9 10
D A B A A B C C D A
11 12 13 14 15 16 17 18 19 20
C B B C B C A C D A
21 22 23 24
D D C B

CatalyseR Eduventures (India) Pvt. Ltd.


APPLICATION OF DERIVATIVES 63

1.10. JEE (ADVANCED) CORNER

1. For a twice differentiable function f(x), g(x) is defined as g (x)  (f '(x) 2  f '(x)) f(x) on [a,
e]. If for a  b  c  d  e, f (a)  0, f (b)  2, f (c)  1, f (d)  2 , f(e) = 0 then find the
minimum number of zeros of g(x). (2006)

2. f(x) is cubic polynomial with f(2) = 18 and f (1)  1 . Also f(x) has local maxima at x   1
and f’(x) has local minima at x = 0, then (2006)
(a) The distance between  1, 2  and (af(a)), where x = a is the point of local minima is 2 5

(b) f(x) is increasing for x [1, 2 5]


(c) f(x) has local minima at x = 1
(d) the value of f(0) = 15

 ex , 0  x 1 x
 x 1
3. Let f (x)  2  e , 1  x  2 and g (x)   f (t) dt, x  [1,3] then g(x) has (2006)
 x  e, 2  x  3 0

(a) local maxima at x = 1 + In 2 and local minima at x = e
(b) local maxima at x = 1 and local minima at x = 2
(c) no local maxima
(d) no local minima

4. The tangent to the curve y  e x drawn at the point (c, e c ) intersects the line joining the points

 c  1, e  and  c  1, e 
c 1 c 1
(2007)
(a) on the left of x = c (b) on the right of x = c
(c) at no point (d) at all points

Passage
If a continuous function f defined on the real line R, assumes positive and negative values in
R then the equation f(x) = 0 has a root in R. For example, if it is known that a continuous
function f on R is positive at some point and its minimum value is negative then the equation
f(x) = 0 has a root in R. Consider f (x)  ke x  x for all real x where k is a real constant.

5. The line y = x meets y  ke x for k  0 at (2007)


(a) no point (b) one point
(c) two points (d) more than two points

Nothing is impossible, the word itself says ‘I’m possible’!


64 APPLICATION OF DERIVATIVES

6. The positive value of k for which ke x  x  0 has only one root is (2007)
1
(a) (b) 1 (c) e (d) log e 2
e

7. For k>0, the set of all value of k for which ke x  x  0 has two distinct roots is
(2007)
 1 1  1 
(a)  0,  (b)  ,1 (c)  ,   (d) (0, 1)
 e e  e 

8. Consider the two curves C1 : y 2  4 x, C2 : x 2  y 2  6 x  1  0 . Then, (2008)


(a) C1 and C2 touch each other only at one point
(b) C1 and C2 touch each other exactly at two points
(c) C1 and C2 intersect (but do not touch) at exactly two points
(d) C1 and C2 neither intersect nor touch each other

9. The total number of local maxima and local minima of the function
3
 2  x  , 3  x  1
f (x)   2/3  is (2008)
 x , 1  x  2 
(a) 0 (b) 1 (c) 2 (d) 3

   
10. Let the function g : ( , )    ,  be given by g (u)  2 tan 1 (eu )  . Then, g is
 2 2 2
(2008)
(a) even and is strictly increasing in (0, )
(b) odd and is strictly decreasing in (, )
(c) odd and is strictly increasing in (, )
(d) neither even nor odd, but is strictly increasing in (, )

1
11. For the function f (x)  x cos , x  1 , (2009)
x
(a) for at least one x in the interval [1,  ), f (x  2)  f (x)  2
(b) lim f '(x)  1
x 

(c) For all x in the interval [1,  ), f (x  2)  f (x)  2


(d) f '(x) is strictly decreasing in the interval [1,  )

CatalyseR Eduventures (India) Pvt. Ltd.


APPLICATION OF DERIVATIVES 65

12. The maximum value of the function f (x)  2 x 3  15 x 2  36 x  48 on the set


A  {x | x 2  20  9 x} is (2009)

 p (x) 
13. Let p(x) be a polynomial of degree 4 having extremum at x = 1, 2 and lim 1  2   2 .
x 0
 x 
Then the value of p (2) is (2009)

14. Let f be a real-valued differentiable function on R (the set of all real numbers) such that
f(1) = 1. If the y-intercept of the tangent at any point P(x,y) on the curve y = f(x) is equal to
the cube of the abscissa of P, then find the value of f ( 3) (2010)

15. Let f be a function defined on R (the set of all real numbers) such that
f '(x)  2010(x  2009)(x  2010) 2 (x  2011) 3 (x  2012) 4 for all x  R . If g is a function
defined on R with values in the interval (0, ) such that f (x)  ln(g(x)), for all x  R Then
the number of points in R at which g has a local maximum is (2010)

x 2
16. If f (x)   t t (t  2)(t  3) dt for all x  (0,  ) , then (2012)
0

(a) f has a local maximum at x = 2


(b) f is decreasing on (2, 3)
(c) there exists some c  (0, ) , such that f '(c)  0
(d) f has a local minimum at x = 3
Passage
x
 2(t  1) 
Let f(x) = (1  x)2 sin 2 x  x 2 for all x IR and let g (x)     ln t  f(t) dt for all
1
t 1 
x (1,  )

17. Consider the statements: (2012)


P : There exists some x  R such That f (x)  2 x  2(1  x 2 )
Q : There exists some x  R such that 2 f (x)  1  2 x (1 + x)
(a) both P and Q are true
(b) P is true and Q is false
(c) P is false and Q is true
(d) both P and Q are false

Nothing is impossible, the word itself says ‘I’m possible’!


66 APPLICATION OF DERIVATIVES

18. Which of the following is true? (2012)


(a) g is increasing on (1,  )
(b) g is decreasing on (1,  )
(c) g is increasing on (1, 2) and decreasing on (2,  )
(d) g is decreasing on (1, 2) and increasing on (2,  )

19. Let f : IR  IR be defined as f (x) | x |  | x 2  1| . The total number of points at which f


attains either a local maximum or a local minimum is ____. (2012)

20. Let p(x) be a real polynomial of least degree which has a local maximum at x = 1 and a local
minimum at x = 3. If p(1) = 6 and p(3) = 2, then p’(0) is____. (2012)

21. A rectangular sheet of fixed perimeter with sides having their lengths in the ratio 8 : 15 is
converted into an open rectangular box by folding after removing squares of equal area from
all four corners. If the total area of removed squares is 100, the resulting box has maximum
volume. Then the lengths of the sides of the rectangular sheet are (2013)
(a) 24 (b) 32 (c) 45 (d) 60

Passage
Let f :[0,1]  R (the set of all real numbers) be a function. Suppose the function f is twice
differentiable, f (0)  f(1)  0 and satisfies f” (x) 2 f '(x)  f (x)  e x , x [0,1] .

22. Which of the following is true for x < x < 1? (2013)


1 1
(a) 0  f (x)   (b)   f (x) 
2 2
1
(c)   f (x)  1 (d)   f (x)  0
4

1
23. If the function e  x f (x) assumes its minimum in the interval [0, 1] at x  , which of the
4
following is true? (2013)
1 3 1
(a) f '(x)  f(x),  x  (b) f '(x)  f (x), 0  x 
4 4 4
1 3
(c) f '(x)  f (x), 0  x  (d) f '(x)  f (x),  x  1
4 4

CatalyseR Eduventures (India) Pvt. Ltd.


APPLICATION OF DERIVATIVES 67

x2 y2
24. A vertical line passing through the point (h, 0) intersects the ellipse  1 at the point P
4 3
and Q. Let the tangents to the ellipse at P and Q meet at the point R. If  (h) = area of the
8
triangle PQR, 1  max (h) and  2  max  (h), then 1  8 2  (2013)
1/ 2  h 1 1/ 2 h1 5

25. Let f : R  R and g : R  R be respectively givne by f (x) | x | 1 and g (x)  x 2  1 .


max { f (x), g(x)} if x  0
Define h : R  R by h(x)   . Then number of points at which
 min { f (x), g(x)} if x  0
h(x) is not differentiable is ___________. (2014)
2 2
26. The slope of the tangent to the curve  y  x 5   x 1  x 2  at the point (1,3) is___________.
(2014)

27. Let f, g:  1, 2   be continuous functions which are twice differentiable on the interval
 1, 2 . Let the values of f and g at the points 1, 0 and 2 be as given in the following table:
x  1 x0 x2
f(x) 3 6 0
g(x) 0 1 -1

In each of the intervals  1, 0 and (0, 2) the function (f – 3g)’’ never vanishes. Then the
correct statement(s) is (are) (2015)
(a) f '  x   3g '  x  0 has exactly three solutions in  1,0   0, 2
(b) f '  x   3g '  x  0 has exactly one solution in  1,0
(c) f '  x   3g '  x  0 has exactly one solution in (0, 2)
(d) f '  x   3g '  x  0 has exactly two solution in  1,0 and exactly two solutions in (0, 2)

192 x3 1 1
28. Let f '  x   for all x   with f    0 . If m   f  x  dx  M ,then the possible values of
2  sin 4 x 2 t/2

m and M are (2015)


1 1
(a) m  13, M  24 (b) m  , M 
4 2
(c) m   11, M  0 (d) m  1, M  12

Nothing is impossible, the word itself says ‘I’m possible’!


68 APPLICATION OF DERIVATIVES

29. A cylindrical container is to be made from certain solid material with the following
constraints: It has a fixed inner volume of V mm3, has a 2 mm thick solid wall and is open at
the top. The bottom of the container is a solid circular disc of thickness 2 mm and is of radius
equal to the outer radius of the container. If the volume of material used to make the
container is minimum when the linner radius of the container is 10 mm, then the value of
V
is (2015)
250 

30. Let f : R   0,   and g : R  R be twice differentiable functions such that f  and g are
continuous functions on R. Suppose f   2   g  2   0 , f   2   0 and g   2   0 . If
f  x g  x
lim  1, then (2016)
x 2 f  x g  x
(a) f has a local minimum at x  2 (b) f has a local maximum at x  2
(c) f   2   f  2  (d) f  x   f   x   0 for at least one x  R

COMPREHENSION
Answer Q.31, Q. 32 & Q. 33 by appropriately matching the nformation given in the three columns
for the following table.
Let f  x   x  log e x  x log e x, x   0,   .

 Column 1 contains information about zeros of f  x  , f   x  and f   x  .

 Column 2 contains information about the limiting behavior of f  x  , f   x  and f   x  at


infinity.
 Column 3 contains information about increasing/decreasing nature of f  x  and f   x  .
Column 1 Column 2 Column 3
(I) f  x   0 for some x  1, e 2  (i) lim f  x   0 (P) f  is increasing in (0, 1)
x 

(II) f   x   0 for some x  1, e  (ii) lim f  x    (Q) f  is decreasing in  e, e 2 


x 

(III) f   x   0 for some x   0,1 (iii) lim f   x    (R) f  is increasing in (0,1)


x 

(IV) f   x   0 for some x  1, e  (iv) lim f   x   0 (S) f  is decreasing in  e, e 2 


x 

31. Which of the following options is the only CORRECT combination? (2017)
(a) (1) (i) (P) (b) (IV) (iv) (S)
(c) (III) (iii) (R) (d) (II) (ii) (Q)

CatalyseR Eduventures (India) Pvt. Ltd.


APPLICATION OF DERIVATIVES 69

32. Which of the following options is the only INCORRECT combination? (2017)
(a) (III) (i) (R) (b) (I) (iii) (P)
(c) (II) (iii) (P) (d) (II) (iv) (Q)

33. Which of the following options is the only CORRECT combination? (2017)
(a) (I) (ii) (R) (b) (II) (iii) (S)
(c) (III) (iv) (P) (d) (IV) (i) (S)

34. If f : R  R is a twice differentiable function such that f   x   0 for all x  R, and

1 1
f    , f 1  1 then : (2017)
2 2
(a) f  1  1 (b) f  1  0
1 1
(c)  f  1  1 (d) 0  f  1 
2 2

cos  2 x  cos  2 x  sin  2 x 


35. If f  x    cos x cos x  sin x , then : (2017)
sin x sin x cos x

(a) f   x   0 at exactly three points in   ,  

(b) f   x   0 at more than three points in   ,  

(c) f  x  attains its maximum at x  0

(d) f  x  attains its minimum at x  0

36. If f : R  R is a differentiable function such that f   x   2 f  x  for all x  R and

f  0   1, then : (2017)

(a) f  x  is decreasing in  0,   (b) f   x   e 2x in  0,  

(c) f  x  is increasing in  0,   (d) f  x   e 2 x in  0,  



Nothing is impossible, the word itself says ‘I’m possible’!


70 APPLICATION OF DERIVATIVES


ANSWER KEY

1 2 3 4 5 6 7 8 9 10
6 BC AB A C A A B C C
11 12 13 14 15 16 17 18 19 20
BCD 7 0 9 1 ABCD C B 5 9
21 22 23 24 25 26 27 28 29 30
AC D C 9 3 8 BC D 4 AD
31 32 33 34 35 36
D A B A BC CD

CatalyseR Eduventures (India) Pvt. Ltd.

You might also like